You are on page 1of 127

目 录

1. Figure Classification ............................................................................... 1

2. Figure Matrices ..................................................................................... 58


1. Figure Classification
1. Explanation
The correct answer is 5.

All three shapes have the same border and fill colors, blue.
Choices 1, 2, 3 and 4 do not have the same border and fill colors as the three given
shapes.
Hence, the 5th choice is the correct answer.

2. Explanation
The correct answer is 3.

The three shapes consist of one smaller shape inside another bigger shape and one of
the two shapes is filled in while the other of the two shapes is not filled in. Therefore,
the correct answer will be a smaller shape inside a bigger shape and only one of the
two shapes will be filled in. Choices 1 and 4 have both shapes not filled in. Choices 2
and 5 have both shapes filled in.

3. Explanation
The correct answer is 5.

In this question you can see that all figures consist of three identical shapes and the
three of them have a common intersection point.
In answer choice 5 there are three identical arrows that all intersect at the same point.
Answer choice 2 can be easily eliminated since it consists of four identical shapes
instead of three.
Answer choices 1, 3, and 4 are incorrect because they do not have one single point
where all three shapes intersect.

4. Explanation
The correct answer is (D).
Each picture includes a shape with a vertical line going through it. This seems like
enough to answer the question, but both answers (A) and (B) have vertical lines going
through them. Notice that the line is a symmetry line which divides the shape into two
equal, mirrored halves, making Answer (D) correct since it is the only answer choice
that contains a symmetry line.

5. Explanation
The correct answer is 2.

The border line and the fill color match in each of the three shapes.
The only option in which the border and fill color are the same is 2 and therefore it is
the correct answer.

1
6. Explanation
The correct answer is 1.

In this question, we can see that:


• The three figures are squares divided into nine smaller parts.
• In each figure, there are three Xs and one circle.
• In each figure, the three Xs are organized in a sequence of a straight or a
diagonal line.
• In each figure, the circle is located in one of the bigger square’s corners.

In the 1st answer choice, we can see a square divided into nine smaller parts. This
square contains three Xs and one circle. We can see that the Xs are organized in
a sequence of a straight line and that the circle is located in one of the bigger
square’s corners.

Therefore, the 1st answer choice is the correct answer.

7. Explanation
The correct answer is 2

In this question the three figures are all shapes that have a blue square in one of the
corners. The only answer choice that is like this is the 2nd choice. The 1st choice is
incorrect as it contains a white square inside. The 3rd choice is also incorrect as it
contains a triangle inside and not a square. Finally, the 4th and 5th choices are
incorrect as the squares they contain are not in a corner.

Therefore, the 2nd choice is the correct answer.

8. Explanation
The correct answer is 2.

All three shapes are arrows that point in multiple directions.


Therefore, the correct answer will be an arrow that points in multiple directions.
Choices 1, 3, 4 and 5 point in only one direction, thus 2, which point in three different
directions, is the correct answer.

9. Explanation
The correct answer is 4

In this question all three figures are made with the following three colors: red, green
and blue. The only answer choice that is like this is the 4th choice. The rest of the
choices contain at least one different color. The 1st choice contains yellow, the 2nd
contains purple and gold, the 3rd contains orange, and the 5th contains purple and
yellow.

2
Therefore, the 4th choice is the correct answer.

10. Explanation
The correct answer is 4.

The three given shapes all have six sides. Therefore, the correct answer will be a
six-sided shape.
Choices 1 and 5 have four sides. Choice 2 has eight sides. Choice 3 has three sides.
Hence, 4 is the correct answer.

11. Explanation
The correct answer is 1

In this question the three figures all contain a shape with curved sides inside a shape
that only has straight sides. Notice that the inner and outer shapes are always different
colors. The only answer choice that is like this is the 1st choice. The 2nd choice is
incorrect because the inner shape is straight and outer shape is curved while the
opposite should be true. The 3rd choice is incorrect as both the inner and outer shapes
are the same color. The 4th choice is incorrect as both of the shapes have curved sides.
Finally, the 5th choice is incorrect as the both of the shapes have only straight sides.

Therefore, the 1st choice is the correct answer.

12. Explanation
The correct answer is 2.

Each of the three figures on the top contains four different colors. Therefore, the
correct answer will be a shape that contains four different colors.
Choices 1, 4, and 5 only contain only three different colors and thus can be ruled out.
Choice 3 contains only two colors and therefore can also be ruled out.
Choice 2 contains four different colors: purple, orange, green, and white – following
the rule. Therefore, it is the correct answer.

13. Explanation
The correct answer is 3.

In this question the three figures are all shapes that are filled with a striped pattern
inside a colored circle. Notice that the stripes are all diagonal from the upper-left to
the lower-right. The only answer choice that is like this is the 3rd choice. The 1st and
4th choices are incorrect as they have stripes that are diagonal in the opposite
direction (from the lower-left to the upper-right). The 2nd choice is also incorrect as
it has horizontal stripes. And the 5th choice is incorrect as it has vertical stripes.

Therefore, the 3rd choice is the correct answer.

3
14. Explanation
The correct answer is 2.

In this question, we can see that:


• All three figures consist of shapes divided into four parts.
• The symbols shared by each of the three figures are the smiley faceand the sun.
These symbols are located in opposite places inside the figures.

The 2nd answer choice is the only one containing a smiley face and a sun located in
opposite places.

Therefore, the 2nd choice is the correct answer.

15. Explanation
The correct answer is 3.

The three shapes all share the same four colors: dark blue, red, green and orange.
Therefore, the correct answer will be a shape that shares these same four colors.
Choice 1 has purple and light blue in place of red and dark blue.
Choice 2 has light blue, black and purple in place of orange, green and red.
Choice 4 has light blue twice and black in place of dark blue, orange and green.
Choice 5 has light blue and purple in place of orange and dark blue.

Hence, the only choice that matches is 3 and it is the correct answer.

16. Explanation
The correct answer is 5.

Each of the given pictures is made by using exactly 3 lines.


Answer 5 is the only answer made using 3 lines, and therefore it is the correct
answer.

17. Explanation
The correct answer is 2.

In this question, we can see that all three figures consist of an outer circle with two
shapes inside. The outer circle is filled with light blue that fades out from the
upper-right to the lower-left. The only answer choice that follows this pattern is the
2nd one.

Therefore, the 2nd figure is the correct answer.

4
18. Explanation
The correct answer is 2.

In the question, each figure consists of a large, outer shape with an inner, smaller
shape.
In each figure (in the top row), the inner shape is different from the large, outer shape,
therefore we can eliminate answer choices 3 and 5.
Additionally, in each of the figures one of the shapes (either inner or outer shape) is
white. Therefore, we can eliminate answer choices 1 and 4.
Thus, the correct answer is 2, a large, outer white triangle and an inner, smaller purple
circle. You may also notice that each diagram contains a circle.

19. Explanation
The correct answer is 5.

All three figures in this question contain shapes divided (approximately) into
one-third (1/3) and two-thirds (2/3) parts.
The only answer choice which follows this rule is the 5th choice.
The 1st, 2nd, and 3rd choices are all divided evenly into halves, so they are incorrect.
The 4th choice is clearly not divided in the same way as the shapes in the question, as
the smaller part is much smaller and the larger part is much larger.
Therefore, the 5th choice is the correct answer.

20. Explanation
The correct answer is 3.
In this question, you can see that:

• Each of the three figures consists of two similar shapes, one on top of the other.
• The top shape is similar to the bottom shape, only rotated 90 degrees clockwise.

In the 3rd answer choice, you can see two similar shapes (trapezoids), one on top of
the other. In addition, the top trapezoid is similar to the bottom one, only rotated 90
degrees clockwise.

Therefore, the 3rd answer choice is the correct answer.

The first answer choice shows two shapes side by side instead of one on top of the
other, and therefore it cannot be correct.

The second answer choice has one shape on top, but it has only been rotated through
45 degrees instead of 90 degrees, so it cannot be correct.

The fourth answer choice shows two shapes, where the top has been rotated through
180 degrees instead of 90 degrees, so it cannot be correct.

5
In the fifth answer choice, the top shape has been rotated 90 degrees anticlockwise so
the top arrow is pointing west instead of east. Therefore, it cannot be correct.

21. Explanation
The correct answer is 1.

The three given shapes are made to look like they take up space in three dimensions.
Therefore, this will be true of the correct answer as well.
Choice 1 is the only one that is made to look like it takes up space in three dimensions,
while all the others appear to take up space in 2 dimensions. Hence, 1 is the correct
answer.
*Note that the first shape in the question is a two-dimensional shape, yet it is indeed
bent into a third dimension, so it takes up space in three dimensions.

22. Explanation
The correct answer is 1.

All three figures in this question are comprised of:

• An outer shape made up of curved lines only.


• Two identical black inner shapes made up of straight lines only.

The only answer choice which follows these rules is the 1st one. The outer shape is
made up of curved lines only, and the two black inner shapes (hexagons) are made
up of straight lines only.

Therefore, the 1st choice is the correct answer.

23. Explanation
The correct answer is 3.
The first shape is a big four-sided shape (square) with a small three-sided shape
(triangle) inside. The second shape is a big six-sided shape (hexagon) with a small
five-sided shape (pentagon) inside. The third shape is an eight-sided shape (octagon)
with a small seven-sided shape (heptagon) inside.
Therefore, the correct answer will be a big shape with a smaller shape inside that has
one less side than the big shape.
In the 1st answer choice, the small shape has one more side than the big shape.
In the 2nd answer choice, the small shape has three less sides than the big shape.
In the 4th answer choice, the big shape does not have any sides.
In the 5th answer choice, the small shape has two less sides than the big shape.

24. Explanation
The correct answer is 5.
In this question, we can see that all three figures in the top row consist of three

6
identical shapes that share a common area:

Similarly, in the 5th answer choice, we can see that all three circles share a common
area:

The rest of the answer choices do not contain an area that is shared by all three
shapes.
Therefore, the 5th answer choice is the correct answer.

25. Explanation
The correct answer is 4.

All the arrows have black borders. Therefore, the correct answer will be the arrow
with a black border.
Choices 1 and 5 have blue borders. Choice 2 has an orange border. Choice 3 has a red
border.
Hence, 4 is the only choice that matches the pattern and it is the correct answer.

26. Explanation
The correct answer is 2.

In this question, the three figures are filled with a stripe pattern. The only answer
choice that is like this is the 2nd choice.
The 1st and 4th choices are incorrect as they are filled with a dotted pattern.
The 3rd choice is incorrect as it is filled with a brick wall pattern.
Finally, the 5th choice is incorrect as it is filled with a checkered pattern.
Therefore, the 2nd choice is the correct answer.

7
27. Explanation
The correct answer is 3.

All three figures in this question contain a circle with three shapes in one of its
quarters:
• A circle at the top,
• A triangle on the bottom left, and –
• A square on the bottom right.

We can immediately eliminate answer choice 2 because it contains shapes in more


than one quarter.
We can also eliminate answer choice 4 because it contains only two shapes and not
three.
Answer choice 5 can be eliminated because it contains two squares and no circle.
Although answer choice 1 contains the right shapes, they are in the wrong order, so it
can be eliminated.
The only answer choice which follows all rules is the 3rd choice.
Therefore, the 3rd choice is the correct answer.

28. Explanation
The correct answer is 4.
Each image contains 2 identical figures that mirror each other.
Image 5 is eliminated because it does not contain two identical figures.
Images 1, 2, and 3 are identical but do not mirror each other.
Image 4 matches the rules mentioned above, therefore, it is the correct answer.

29. Explanation
The correct answer is 4 because:
 all the images contain an odd number of lines, and
 all the images have a different colour from each other.
Images 1, 3 and 5 are eliminated because they contain figures with an even number of
lines.
Image 2 contains an odd number of sides but shares the same colour as another figure
already in the series (the pentagon)
Image 4 matches the rules mentioned above, therefore, it is the correct answer.

30. Explanation
The correct answer is 5.
Each picture includes two shapes; an outer shape patterned with diagonal lines, and
another inner shape.
In addition, the inner shape has one side less than the outer shape.
Answer 5 fits the pattern and therefore it is the correct answer.
Answers 1 and 4 are wrong because the backgrounds are white rather than the inner
shape.

8
Answer 2 is wrong because the inner shape has the same number of edges as the
shape on the outside.
Answer 3 is wrong because the outer shape (the triangle) has one less edge than the
inner shape (the parallelogram), instead of the other way around.

31. Explanation
The correct answer is 1.

The given pictures have a grid pattern, thus answers 2 and 4, which have different
patterns, can be ruled out. 2 has a pattern of short vertical lines, and 4 has a pattern of
diagonal lines.
In addition, all the given pictures include a shape with seven vertices. The only
remaining answer with seven vertices is 1, thus it is the correct answer. Answer 3 has
six vertices, and answer 5 has 12 vertices.

32. Explanation
The correct answer is 4.
In all the images, the arrow touches the other figure with only one edge.
In images 1, 2, 3, and 5, all the arrows are touching the figures with 2 edges.
Only in image 4, the arrow touches the figure with one edge, and therefore, it is the
correct answer.

33. Explanation
The correct answer is 2.

The three shapes all have seven sides. Therefore, the correct answer will be a
seven-sided shape.
Choice 1 has three sides. Choice 3 has four sides. Choice 4 doesn’t have any sides.
Choice 5 has seventeen sides.
Hence, 2 is the correct answer.

34. Explanation
The correct answer is 3.

None of the three given shapes is filled with color.


Answers 1, 2, 4, and 5 are all filled with color.
Hence, 3 is the correct answer.

35. Explanation
The correct answer is 2.

In this question you can see that:


- All figures consist of three shapes: one black circle and two identical, big, white

9
shapes, when one of them covers part of the other.
- The shape that covers part of the other shape is the shape that is closer to the circle.

In answer choice 2, the shape that covers the other shape is located next to the circle.
Therefore, this is the correct answer.

Answer choices 1 and 4 are incorrect because the shape that covers the other is further
away from the circle.
Answer choices 3 and 5 are incorrect because both shapes are the same distance from
the circle.

36. Explanation
The correct answer is 4.

In this question you can see that:


- All figures consist of three different shapes: a triangle pointing down, a square, and
a circle.
- Each shape is filled differently and consistently: the triangle is white, the square is
black, and the circle is checkered.

Answer choice 4 consists of a white triangle pointing down, a checkered circle, and a
black square. Therefore, this is the correct answer.

You can eliminate answers 1, 3, and 5 immediately as the triangle is pointing up


instead of down.
You are now left with answers 2 and 4. In answer 2, the square is checkered and the
circle is black, while in each of the top figures the circle is checkered and the square is
black. Therefore, answer 2 is incorrect and you are left with answer 4.

37. Explanation
The correct answer is 2.

All three shapes have a much longer height than width.


Choices 1, 4, and 5 have approximately equal height and width. Choice 3's width is
longer than its height.
Hence, 2 is the correct answer.

38. Explanation
The correct answer is 5.

All three arrows are the same size. Choices 1, 2, 3, and 4 are different in their sizes
than the three given arrows. In addition, the arrows are rotating in a clockwise
direction, so the missing arrow should be rotated 90 degrees clockwise from the
previous image, like in answer 5.

10
Hence, 5 is the correct answer.

39. Explanation
The correct answer is 1.

In this question the three figures are ribbons with two triangular points at each end.
The only answer choice that is like this is the 1st choice. The 2nd, 4th and 5th choices
are incorrect as they are shaped like scrolls and not like ribbons. The 3rd choice is
incorrect as it is wavy and not folded in the same way. All answer choices except the
first do not have two triangular points at the end.

Therefore, the 1st choice is the correct answer.

40. Explanation
The correct answer is 4.

In this question you can see that all figures have four arrows—two are pointing up
and two are pointing down. Additionally, you can see that all figures consist of a
closed outer shape but there is no common rule regarding the outer shape’s features.
Since all the answer choices also consist of a closed outer shape, you should disregard
this rule and take into consideration only the arrows when looking for the correct
answer.

Answer choice 4 has two arrows pointing up and two arrows pointing down and
therefore it is the correct answer.

Answer choice 1 is incorrect because all four arrows are pointing down.
Answer choice 2 is incorrect because it has three arrows pointing up and one arrow
pointing down.
Answer choice 3 is incorrect because it has only three arrows.
Answer choice 5 is incorrect because it has one arrow pointing up and three arrows
pointing down.

41. Explanation
The correct answer is 5.

In this question all three figures are shapes with three squares, some appearing in 3D,
with one shape in each square.
The squares in all three figures contain a sun, a heart and a banner.

Answer choice 1 is a shaped like a can and does not contain three squares.
Answer choice 2 does contain three squares, but the banner does not look like the one
in the figures above.

11
Answer choice 3 does not contain three squares with a shape in each one.
Answer choice 4 contains an octagon and not three squares.

Therefore, the 5th choice is the correct answer.

42. Explanation
The correct answer is 3.

In this question you can see that:

- All figures are quadrilateral (they all have four straight sides and four sharp
corners).
- In each figure there is one corner shaded blue. Answer choice 3 is a quadrilateral
and its top left corner is shaded. Therefore, it is the correct answer.

Answer choices 1, 2, and 4 can be eliminated because they are not quadrilaterals
(answer choice 1 has no straight sides, answer choice 2 has rounded corners—not
sharp ones, and answer choice 4 has 5 sides).

Now you are left with answer choices 3 and 5, and should look for another rule in
order to find the correct answer. You can see that the difference between these answer
choices is the shading. In answer 3 the shading is in one of the corners, and in answer
5 the shading is along the side. When looking back at the three top figures, you can
see that in all of them the shading is on a corner. Therefore, you can eliminate answer
choice 5 as well.

43. Explanation
The correct answer is 3

In this question the three figures are all squares with a small shape in the upper left
corner. Notice that the inner and outer shapes are always different colors. The only
answer choice that is like this is the 3rd choice. The 1st choice is incorrect as both the
inner and outer shapes are the same color. The 2nd choice is incorrect because the
inner shape is too large. The 4th choice is incorrect as the outer shape is not a square.
Finally, the 5th choice is incorrect as the inner shape is at the right as opposed to the
left upper corner.

Therefore, the 3rd choice is the correct answer.

44. Explanation
The correct answer is 5.

In this question the three figures each contain a series of the different shapes that fit
one inside the other. Notice that the inner and outer shapes are always the same color

12
but the middle shape is a different color. The only answer choice that is like this is the
5th choice. The 1st choice is incorrect as all the shapes are the same (they are all
squares). The 2nd choice is incorrect because the middle shape is not a different color
than the rest of the shapes. The 3rd choice is incorrect as the inner and outer shapes
are not the same color. Finally, the 4th choice is incorrect because the inner shape
does not fit entirely inside the middle shape.

Therefore, the 5th choice is the correct answer.

45. Explanation
The correct answer is 3.

In this question you can see that:

- Each of the above three figures has an outer shape with straight lines only.
- Each of the figures contain two inner circles.
- In each figure, the two inner circles have the same filling.

Answer choice 3 is the correct answer as its outer shape is comprised of straight lines
only, and as it contains two inner circles with the same filling.

Answer choices 1 and 4 are incorrect as they are comprised also of curved lines.
Answer choice 2 is incorrect as its inner circles do not have the same filling.
Answer choice 5 is incorrect as it contains three inner circles instead of two.

46. Explanation
The correct answer is 1.

In this question you can see that:


- All figures include a black outer shape and three white inner smaller shapes.
- In all figures the outer shape has straight sides and sharp corners.
- In all figures there are three inner shapes; two of them have straight sides and sharp
corners, and one is a full circle.

Answer choice 1 has an outer shape with straight sides and sharp corners, and its inner
shapes consists of two shapes that have straight sides and sharp corners, and one full
circle. Therefore, it is the correct answer.

Answer choices 2 and 4 can be eliminated immediately since their outer shape is a
circle.
Answer choice 3 has two inner circles while all the figures on top consist of only one
circle. Therefore, you can eliminate this answer as well.
Now you are left with answers 1 and 5, and you need to find another rule to get to the
correct answer. You can see that there are two differences between the two answers:

13
- Answer 1 has two identical inner shapes while in answer 5 all the inner shapes are
different.
- Answer 1 has a full circle while answer 5 has a ring-shaped circle.
When you compare the figures in the answers again to the three figures on top, you
can see that the circle in all three figures on top is a full circle and there is no
ring-shaped circle in any of them. Therefore, you can also eliminate answer 5, and
you are left with answer 1.

47. Explanation
The correct answer is 4.

In this question you can see that each shape is further cut into two smaller parts which
are attached to the shape’s boundaries. In addition, each shape contains an identical
inner shape – the triangle on the left contains a smaller triangle, the circle in the
middle contains a smaller circle, and the hexagon on the right contains a smaller
hexagon. Notice that the inner shape and the cut parts all have the same color or
pattern. For example, those parts in the left figure all have a checkered pattern.

Answer 4 is correct as the teardrop contains a smaller inner teardrop, and two cut
parts. The inner teardrop and the cut parts are all colored black.

Answer 1 is incorrect as the cut part on the left of the rectangle is not colored black.
Answer 2 is incorrect as the inner ‘L’ shape is not filled with a checkered pattern.
Answer 3 is incorrect as the inner shape (a triangle) is not the same as the outer shape
(a trapezoid).
Answer 5 is incorrect as it has only one cut part instead of two.

48. Explanation
The correct answer is 3.

In this question, we can see that:


 All three figures consist of an outer shape with a smaller inner shape. The
smaller inner shape is similar to the outer shape.
 All shapes are comprised of straight lines only.
 The outer shape is colored with light blue that fades out from bottom to
top.

The 1st choice is incorrect as it is comprised of curved lines.


The 2nd choice is incorrect as it is also comprised of curved lines.
The 4th choice is incorrect as the outer shape is colored with light blue that fades out
from right to left.
The 5th choice is incorrect as the inner shape is colored instead of the outer shape.

Therefore, the 3rd choice is the correct answer.

14
49. Explanation
The correct answer is (C).
In the pictures above there is a trapezoid with three circles above it. The circles’
patterns (or colors) correspond to the segments of the trapezoid below it which are
patterned (or colored) in the same way. In addition, every color or pattern repeats
itself only once in each trapezoid segment.
Answer (C) is correct, as it is the only answer that matches the pattern. It has circles
patterned in the same way as the segments below them, and all the patterns are
different from one another.
In answers (A), (B), and (D) the pattern of the circles does not match the segments
below them, and in answer (E) there are two segments with the same pattern (black
color).

50. Explanation
The correct answer is 4.

The three shapes all have an even number of sides. Therefore, the correct answer will
have an even number of sides. Choice 1 has five sides, choices 2 and 5 have seven
sides, and choice 3 doesn’t have any sides.

The only choice left with an even number of sides is choice 4, which is the correct
answer.

51. Explanation
The correct answer is 3.

In this question, the three figures above have straight sides and are filled in
with diagonal lines that slant up to the right.

The third answer choice follows these conditions: it is a shape that consists of only
straight sides, and is filled in with diagonal lines slanted up to the right. Therefore,
this is the correct answer.

The first, fourth, and fifth answer choices are incorrect as they all consist of curved
lines as well as straight lines.
The second answer choice is incorrect as although it consists of straight lines only, it
is filled in with diagonal lines slanted up to the left instead of being slanted up to the
right.

Tip: If you find it difficult to infer the rule connecting the three figures above, it
might be useful to look at the answer choices to see if there are any unusual figures. In
this question, you can see that only answer choices 2 and 3 have straight lines, so you
can check if this is the correct rule.

15
52. Explanation
The correct answer is 2.

In this question you can see that:


- All figures consist of two shapes: a black circle and a shape with straight lines and
sharp corners.
- In all figures, at least some of the corners of the shape with the straight lines
intersect with the circle's outer line.

In answer choice 2 there is a black circle and a shape that has straight lines and sharp
corners, and some of the corners intersect with the circle’s outer line.

Answer choice 3 can immediately be eliminated as its circle is white, and each of the
circles above are black.
You can also eliminate answer choice 5 because its straight-line shape does not
intersect with the circle’s outer line at all.
Looking carefully, answers 1 and 4 can be eliminated since the corners of the shape
with straight lines never intersect with the circle’s outer line - they only intersect in
the middle of the straight lines.
You are then only left with answer 2, which is the correct answer, as there is a black
circle, and the straight-lined shape intersects with it at some of its corners.
Alternatively, you can solve the problem with the following rules:
- All figures consist of two shapes: a black circle and a shape with straight lines and
sharp corners.
- In all figures, the straight-lighted shape has an even number of sides (the figures in
the question have 4, 10, and 14 sides respectively).
In this case, the only answer choice that features a shape with even sides is answer 2,
so this must be the correct answer.

53. Explanation
The correct answer is 5.

In this question you can see that:


- All figures are circles that have lines on both sides.
- All the figures have the same number of lines on each side.

Answer choice 5 has four lines on both the right side and on the left side of the circle,
and it is therefore the correct answer.

Answer choices 1, 2 and 3 are immediately eliminated since they do not have the
same number of lines on the left and right side of the circle.
After eliminating these answers you are left with answer choices 4 and 5, which have
the same number of lines. Since it seems that answer choice 4 could also be correct
because it has the same number of lines on each side of the circle (zero lines), you

16
must find another rule in order to eliminate one answer. As all the top figures have
lines, the correct answer must have lines and therefore answer 4 is eliminated.

54. Explanation
The correct answer is 1.

In this question all three figures are triangles in various colors each of which contains
a purple circle, a green circle and an orange circle. The only answer choice that is like
this is the 1st choice. The rest of the choices contain one circle that is the wrong color.
In the 2nd choice the orange circle is replaced by a blue circle, in the 3rd choice the
green one is replaced by a red one, the 4th choice has a yellow circle instead of the
purple circle, and the 5th choice contains a blue circle instead of the purple one.

Therefore, the 1st choice is the correct answer.

55. Explanation
The correct answer is (B).
From left to right, each of the shapes moves one step counterclockwise. In addition,
the colors of the shapes alternate between black and white in each picture.
The last picture had a black rhombus in the top right-hand corner. The next picture
should have a white rhombus one step counterclockwise - in the top left-hand corner.
Similarly, since the last picture had a white star in the bottom right-hand corner, the
next picture should have a black star in the top right-hand corner.
Therefore, answer (B) is correct, as it is the only answer that fits the pattern. The
shape moves once again counterclockwise, and their colors are reversed compared to
the previous picture.

56. Explanation
The correct answer is 1.

In this question you can see that all figures have an odd number of lines.

Answer choice 1 has five lines, which is an odd number, and therefore it is the correct
answer.

Answer choices 2 and 4 are incorrect because they have six lines, which is an even
number of lines.
Answer choice 3 and 5 are incorrect because they have four lines, which is an even
number of lines.

57. Explanation
The correct answer is 3.

In this question you can see that all figures consists of three shapes that are identical

17
but in three different sizes.

The cloud shape in answer choice 3 is repeated three times in three different sizes.
Therefore, it is the correct answer.

Answer choice 1 is incorrect because the big sized square is repeated twice and
therefore the figure has only two sizes.
Answer choices 2 and 4 are incorrect because the shape is repeated three times in the
same size.
Answer choice 5 is incorrect because it consists of only two shapes.

58. Explanation
The correct answer is 5.

The first shape is a big five-sided shape (pentagon) with a small six-sided shape
(hexagon) inside. The second shape is a seven-sided shape (heptagon) with a small
eight-sided shape (octagon) inside. The third shape is a big three-sided shape (triangle)
with a small four-sided shape (square) inside.
Therefore, the correct answer will be a big shape with a smaller shape inside that has
one more side than the big shape.

In the 1st answer choice, the small shape has two less sides than the big shape.
In the 2nd answer choice, the big shape does not have any sides.
In the 3rd answer choice, the small shape has three less sides than the big shape.
In the 4th answer choice, the small shape has one less side than the big shape.

59. Explanation
The correct answer is 1

In this question all of the edges of the shapes in the three figures are straight. The only
answer choice that is like this is the 1st choice. The rest of the choices have one or
more rounded corners.

Therefore, the 1st choice is the correct answer.

60. Explanation
The correct answer is 5

In this question the shapes in all three figures are equilateral (this means that all the
sides of each shape are the same length). The only answer choice that is like this is the
5th choice. The rest of the choices have one or more sides that are a different length
than the rest.

18
Therefore, the 5th choice is the correct answer.

61. Explanation
The correct answer is 3

In this question the three figures are all shapes that have a green triangle inside on one
of the sides of the shape. The only answer choice that is like this is the 3rd choice.
The 1st, 2nd and 4th choices are incorrect as the triangle is in a corner of the outer
shape and not on one of the sides. The 5th choice is also incorrect as the triangle it
contains is white and not green.

Therefore, the 3rd choice is the correct answer.

62. Explanation
The correct answer is 1

In this question all three figures are shapes that are divided by a horizontal and a
vertical line inside. The only answer choice that is like this is the 1st choice. The rest
of the choices are divided by diagonal lines.

Therefore, the 1st choice is the correct answer.

63. Explanation
The correct answer is 3

In this question the three figures are all four sided shapes filled with a dotted pattern.
The only answer choice that is like this is the 3rd choice. The 1st, 2nd and 5th choices
are incorrect as they are not four sided. The 2nd and 4th choices are also incorrect as
they are filled with a broken line pattern as opposed to a dotted pattern.

Therefore, the 3rd choice is the correct answer.

64. Explanation
The correct answer is 5

In this question the three figures are all different colored octagons with a narrow plus
sign in the middle which is filled with various patterns. Notice that the octagon itself
is always a solid color. The only answer choice that is like this is the 5th choice. The
1st choice is incorrect as it has a wide border around the octagon shape but shouldn't.
The 2nd choice is incorrect because the octagon itself is patterned while the plus sign
isn't and it should be the opposite. The 3rd choice is incorrect as it is a square and not
an octagon. Finally, the 4th choice is incorrect as it has a plus sign in the center that is
too large.

19
Therefore, the 5th choice is the correct answer.

65. Explanation
The correct answer is 3.

All three figures contain a diagonal line that slants upward toward the right. All the
diagonal lines in the question are parallel, meaning they have the same steepness and
point the same way.
The only answer choice that follows this rule is the third choice. The rest of the
options do not contain a similar diagonal line that is parallel to the other diagonal
lines.

Therefore, the third choice is the correct answer.

66. Explanation
The correct answer is 5.

All three figures in this question contain circles with a vertical line that runs through
the middle.
In each half of the circle, we see a smaller shape with a diagonal line that
slants upwards and towards the middle of the outer circle.

The only answer choice which follows this rule is the 5th choice.
Therefore, the 5th choice is the correct answer.

67. Explanation
The correct answer is 1.

All three figures in this question contain shapes divided in half.


The only answer choice which follows this rule is the 1st choice.
Therefore, the 1st choice is the correct answer.

68. Explanation
The correct answer is 5.

At first glance, we see that all three figures in this question are shapes that point
upwards and towards the right.
However, there is more than one answer choice that points upwards and towards the
right.
So, which answer is correct?

Let’s look again at the shapes in the top row. What else do they have in common?
When we look closely, we see that all 3 shapes are comprised of straight lines.

20
The only answer choice which follows this rule is the 5th choice. Since there are
multiple options that point up and to the right, and answer 5 is the only one of the
choices that is composed of only straight lines, we know that option 5 must be the
right answer even though it points to the left. Pointing to the right is not a relevant
factor since it is sufficient that the three shapes above and option 5 are all composed
of only straight lines.

Therefore, the 5th choice is the correct answer.

69. Explanation
The correct answer is 3.

In this question, we can see that:


• All three figures include a cylinder, a rectangle and a triangle
• The shapes are either white, dark or light blue
• Each shape and shading appear only once in each figure

Answer choice 1 can be eliminated because it contains a hexagon instead of a


rectangle.
Answer choice 2 can be eliminated because more than one shape is dark.
Answer choice 4 can be eliminated because more than one shape is white.
Answer choice 5 can be eliminated as well since the rectangle has been replaced with
a different shape.

The only answer choice which follows all the rules is the 3rd choice.

Therefore, the 3rd choice is the correct answer.

70. Explanation
The correct answer is 4.

All three figures in this question contain the same shape only rotated: a circle
divided into three sections with a smaller shape in each section.
Therefore, we need to find an answer choice which also contains the same shape, no
matter how it is positioned.

Answer choices 1, 2 and 3 can be eliminated as they do not contain a circle in the
middle section like the figures in the top row.
When we look more closely at answer choice 5, we see that the square and the
triangle have switched places.

Therefore, we are left with the 4th choice, which is the only correct answer.

21
71. Explanation
The correct answer is 2.

Each of the three figures in this question includes one curved line and
several straight lines.

In answer choice 1 the shape has two curved lines and not one.
The shape in answer choice 3 has two curved lines as well.
The shape in answer choice 4 has no curved lines at all.
The shape in answer choice 5 has only a curved line and no straight lines.

Answer choice 2 is the only one containing straight lines and one curved line.

72. Explanation
The correct answer is 4.

Each of the figures in this question has seven sides and a triangle on the inside.
The only answer choice which follows these rules is the 4th choice.
All of the other answer choices contain shapes with fewer than seven sides, and/or the
inner shape is not a triangle.

Therefore, the 4th choice is the correct answer

73. Explanation
The correct answer is 1.

Each of the three figures in this question contains only one dark triangle and only
one white triangle that are opposite each other.
The only answer choice that follows this rule is the 1st choice.
The 2nd and 3rd answer choices include two dark triangles and no white triangles.
The 4th answer choice does not include a dark triangle.
The 5th answer choice does not include a white triangle.

Therefore, the 1st choice is the correct answer.

74. Explanation
The correct answer is 4.

In this question all three figures are:


• Triangles,
• Which are divided into three smaller triangles…
• …that all meet at a single point.

22
Furthermore, in all three figures, if the figure is rotated in a way that the meeting point
is at the top of the figure:
• The small triangle in the middle is white,
• The small triangle on the right is dark, and -
• The small triangle on the left is striped.

The only answer choice which follows all of these rules is the 4th choice.
Notice that in the 5th answer choice, if the figure is rotated in a way that the meeting
point is at the top of the figure (and not at the bottom as it is in the answer) the dark
small triangle will be on the left, and the striped small triangle will be on the right.
That is the opposite order of what is shown in the series. Thus, the 5th answer choice
is incorrect.

Therefore, the 4th choice is the correct answer

75. Explanation
The correct answer is 4.

All three figures contain a house with:


• A tiled roof,
• One door and –
• Two rectangular windows.

The only answer choice which follows this rule is the 4th choice.

Answer choice 1 has three doors and one window.


Answer choice 2 has only one window and a roof which is not tiled.
Answer choice 3 also has a roof which is not tiled.
Answer choice 5 has circular windows.

Therefore, the 4th choice is the correct answer.

76. Explanation
The correct answer is 5.

All three figures in this question contain a frame with two suns and five clouds.
The only answer choice which follows this rule is the 5th choice.
Answer choice 1 is incorrect because it contains three suns and three clouds.
Answer choice 2 is incorrect because it contains one sun and five clouds.
Answer choice 3 is incorrect because it contains two suns and six clouds.
Answer choice 4 is incorrect because it contains three suns and five clouds.

Therefore, the 5th choice is the correct answer.

23
77. Explanation
The correct answer is 4.

In this question, we can see the following:


All three figures are shapes containing one diagonal inner line.
The inner line divides each shape into two unequal parts.
All three shapes contain a cloud, which is located above the inner line.

In the 4th answer choice, there is a shape with a diagonal inner line, dividing that
shape into two unequal parts. Moreover, this shape contains
a cloud located above the inner line.

The 1st and 2nd answer choices are incorrect since in both cases the inner line is not
diagonal and divides the shape into two equal parts.
The 3rd answer choice is incorrect as the cloud is located below the inner line while it
should be above the line.
The 5th answer choice is incorrect as it contains two inner lines instead of one.

78. Explanation
The correct answer is 2.

In this question, we can see that:


 All three figures contain a shape with two inner lines that meet at a single
point.
 One of the inner lines is straight, while the other is curved.

The 1st answer choice is incorrect as the two inner lines are straight.
The 3rd answer choice is incorrect as this shape contains three inner lines.
The 4th answer choice is incorrect as the two inner lines inside the shape do not meet
at any point.
The 5th answer choice is incorrect as the two inner lines are curved.

In the 2nd answer choice, we can see a shape containing two inner lines that meet at
a single point. One of the lines is straight and the other is curved.

Therefore, the 2nd answer choice is the correct answer.


Was this explanation helpful?

79. Explanation
The correct answer is 5.

In this question, we can see that all three figures in the top row consist of three
identical shapes that share a common area:

24
Similarly, in the 5th answer choice, we can see that all three circles share a common
area:

The rest of the answer choices do not contain an area that is shared by all three
shapes.

Therefore, the 5th answer choice is the correct answer.

80. Explanation
The correct answer is 2.

In this question, we can see that:


 All three figures contain a shape with an inner arrow pointing to one of the
shape’s corners.

25
 In each of the figures, one of the shapes (either the inner arrow or the outer
shape) is white, while the other shape is dark blue.

In the 2nd answer choice, we can see a shape (triangle) with an inner arrow pointing
to one of its corners. In addition, the triangle is dark blue, while the arrow inside it
is white.

Therefore, the 2nd answer choice is the correct answer.

81. Explanation
The correct answer is 3.
All three figures in this question are divided into two equal halves. In addition, the
separating line is a diagonal line slanted up to the left.
The third figure is divided into two equal halves, and the separating line is a diagonal
line which slants up to the left. Thus, this is the correct answer.
The first answer choice is incorrect as the separating line slants up to the right.
The second answer choice is incorrect as the separating line is horizontal when it
should be diagonal.
The fourth answer choice is incorrect as the separating line slants up to the right.
The fifth answer choice is incorrect as the figure is not divided into two equal halves.

82. Explanation
The correct answer is 5.
In this question, all of the three figures above are circles divided into four parts. Each
one of the circles contains one smiley face and one star. The smiley face and the star
are located opposite to each other.
The fifth figure contains one smiley face and one star that are located opposite to each
other. Thus, this is the correct answer.
The first answer choice is incorrect as it does not contain a star.
The second and fourth answer choices are incorrect as the smiley face and the star are
located next to each other instead of opposite each other.
The third answer choice is incorrect as it does not contain a smiley face.

83. Explanation
The correct answer is 1.

All three figures in this question are comprised of:

 An outer shape made up of curved lines only.


 Two identical black inner shapes made up of straight lines only.
The only answer choice which follows these rules is the 1st one. The outer shape is
made up of curved lines only, and the two black inner shapes (hexagons) are made
up of straight lines only.
Therefore, the 1st choice is the correct answer.

26
84. Explanation
The correct answer is 2.
In this question, all three figures above consist of a big shape in the middle, with
the same two smaller shapes, one on the right and one on the left. In addition, notice
that all the figures are aligned, and that the smaller shapes are in the same direction as
the big shape.
The second figure consists of a big pentagon in the middle with two smaller
pentagons on its right and left. Notice that the small pentagons are in the same
direction as the big pentagon, and that the whole figure is aligned. Thus, this is the
correct answer.
The first answer choice is incorrect as the two smaller rectangles are located above
and beneath the big rectangle, instead of being located on its sides.
The third answer choice is incorrect as the small pentagons are pointing downwards
while the big pentagon is pointing upwards, when instead they all should be on the
same direction.
The fourth answer choice is incorrect as the figure is not aligned but rather slants to
the right.
The fifth answer choice is incorrect as the smaller shapes (rectangles) are not the same
as the big shape (the pentagon).

85. Explanation
The correct answer is 1.
In this question, you can see that the three figures above consist of three shapes each,
which are set in a diagonal manner: a circle, a trapezoid, and a star. In addition, in
each figure, the shapes (the circle, the trapezoid, and the star) are either dark
blue, white, or checkered. Notice that there is no match between a specific shape and
a specific color, and that no two shapes in the same figure have the same filling.

The first figure consists of the three correct shapes: the circle, the trapezoid, and the
star. These shapes are correctly set diagonally. In addition, these shapes have the
correct colors: white (the trapezoid), dark blue (the circle), and checkered (the star).
Thus, this is the correct answer.
The second and third answer choices are incorrect as although they consist of the
correct shapes and have the correct colors, the shapes in them are not set diagonally,
and therefore they can be eliminated.
The fourth and fifth answer choices are incorrect as although they have the correct
shapes which are correctly set diagonally, they do not have the correct colors: the
fourth answer choice is not white, and the fifth answer choice is not checkered.
Therefore, these answer choices can be eliminated.

86. Explanation
The correct answer is 3.
In this question, the three figures above have straight sides and are filled in
with diagonal lines that slant up to the right.

27
The third answer choice follows these conditions: it is a shape that consists of only
straight sides, and is filled in with diagonal lines slanted up to the right. Therefore,
this is the correct answer.

The first, fourth, and fifth answer choices are incorrect as they all consist of curved
lines as well as straight lines.
The second answer choice is incorrect as although it consists of straight lines only, it
is filled in with diagonal lines slanted up to the left instead of being slanted up to the
right.
Tip: If you find it difficult to infer the rule connecting the three figures above, it
might be useful to look at the answer choices to see if there are any unusual figures. In
this question, you can see that only answer choices 2 and 3 have straight lines, so you
can check if this is the correct rule.

87. Explanation
The correct answer is 4.
In this question, the three figures above all contain a diagonal line that connects one
of the shape’s corners with one of the shape’s sides. Notice that the line does
not connect two corners.
The shape in the fourth answer choice contains a diagonal line that stretches from its
bottom left corner to the middle of its upper right side. Therefore, this is the correct
answer.
The first answer choice is incorrect as the line in this shape connects two sides and
does not reach a corner.
The second answer choice is incorrect as the line in this shape connects only corners
and does not reach one of the shape’s sides.
The third answer choice is incorrect as there are two lines in this shape while there
should be only one.
The fifth answer choice is incorrect as the line in this shape connects two sides and
does not reach a corner.

88. Explanation
The correct answer is 3.
In this question, the three figures above all contain two inner lines
that intersect inside the shape. Notice that the inner lines divide each figure
into four parts (not necessarily equal).
The third answer choice is a pentagon which contains two inner lines that intersect
inside it. In addition, these lines divide the pentagon into four parts. Therefore, this is
the correct answer.
The first and the second answer choices are incorrect as the two inner lines in these
figures do not intersect.
The fourth answer choice is incorrect as this figure contains three inner lines instead
of two.

28
The fifth answer choice is incorrect as although this figure is divided into four parts, it
contains three inner lines instead of two.

89. Explanation
The correct answer is 1.
In this question, the three figures above are divided into two parts: a white part and
a checkered part. In addition, within the white part, there is a small white
shape which is the same as the outer figure.
The first answer choice is a pentagon divided into a white part and a checkered part.
Within the white part there is a small pentagon, which is the same as the outer figure.
Therefore, this is the correct answer.
The second answer choice is incorrect as the small shape is the same as the shape of
the white part (a trapezoid), while it should be the same as the outer figure (a
hexagon).
The third answer choice is incorrect as the small shape is located within the checkered
part instead of the white part.
The fourth answer choice is incorrect as the small shape (a triangle) is checkered
instead of being white. In addition, the outer figure (except for the inner small triangle)
is white, while it should be both white and checkered.
The fifth answer choice is incorrect as the small shape (a triangle) is different than the
outer figure (a parallelogram).

90. Explanation
The correct answer is 5.
In this question, we can see that:
 The three figures are circles containing two inner shapes, one to each side of
an inner line that cuts the circles in half.
 One of the inner shapes is white, while the other inner shape is black.
 In each figure, the two inner shapes have a different number of sides.
The 1st and 3rd answer choices are incorrect as the two inner shapes in these choices
have the same number of sides.
The 2nd and 4th answer choices are incorrect as the two inner shapes in these choices
are the same color.
The 5th answer choice is a circle containing one black shape and one white shape. In
addition, these two shapes have a different number of sides (pentagon with five
sides and triangle with three sides).
Therefore, the 5th answer choice is the correct answer.

91. Explanation
The correct answer is 4

In this question the three figures are all shapes that have perfect squares in the center.
The only answer choice that is like this is the 4th choice. The 2nd, 3rd and 5th choices
are incorrect as they contain inner shapes that are not square and have the wrong

29
number of sides. The 2nd choice is also incorrect as it contains a four sided shape that
is not a square but a diamond.

Therefore, the 4th choice is the correct answer.

92. Explanation
The correct answer is 4

In this question all three figures are shapes with inverted corners (some of the shape's
corners point into the shape). The only answer choice that is like this is the 4th choice.
The rest of the choices only contain corners that point outward.

Therefore, the 4th choice is the correct answer.

93. Explanation
The correct answer is 1

In this question the three figures all contain a shape with curved sides inside a shape
that only has straight sides. Notice that the inner and outer shapes are always different
colors. The only answer choice that is like this is the 1st choice. The 2nd choice is
incorrect because the inner shape is straight and outer shape is curved while the
opposite should be true. The 3rd choice is incorrect as both the inner and outer shapes
are the same color. The 4th choice is incorrect as both of the shapes have curved sides.
Finally, the 5th choice is incorrect as the both of the shapes have only straight sides.

Therefore, the 1st choice is the correct answer.

94. Explanation
The correct answer is 5

In this question all three figures are shapes that have no inverted corners (all of the
shapes corners point outward). The only answer choice that is like this is the 5th
choice. The rest of the choices contain corners that point inverted corners (that
point into the shape).
Alternatively, another pattern this question presents is a series of shapes in which each
shape has one side less than the one before. The first shape is a green hexagon (6
sides), followed by a purple pentagon (5 sides) and an orange quadrilateral (3 sides).
The next shape in the series, therefore, would be a triangle (3 sides).
Therefore, the 5th choice is the correct answer.

95. Explanation
The correct answer is 2

In this question the three figures are all purple octagons with a narrow plus sign in the

30
middle which is filled with various patterns. Notice that the octagon itself is always
solid purple. The only answer choice that is like this is the 2nd choice. The 1st choice
is incorrect as it has a plus sign in the center that is too large. The 3rd choice is
incorrect as it is a square and not an octagon. The 4th choice is incorrect because the
octagon itself is patterned while the plus sign isn't and it should be the opposite.
Finally, the 5th choice is incorrect as it has a wide border around the octagon shape
but shouldn't.

Therefore, the 2nd choice is the correct answer.

96. Explanation
The correct answer is 4

In this question all three figures are orange circles each of which contains the
following three shapes: a yellow diamond, triangle and circle. The only answer choice
that is like this is the 4th choice. The 1st choice is incorrect as it contains two
diamonds and no triangle. The 2nd and 3rd choices are incorrect as they contain
trapezoids instead of triangles. Finally, the 5th choice is incorrect as it contains four
shapes instead of three (a trapezoid in addition to all of the other shapes).

Therefore, the 4th choice is the correct answer.

97. Explanation
The correct answer is 2

In this question all three figures are triangles each of which contains the following
three shapes: a tilted square, a triangle and a circle. The only answer choice that is like
this is the 2nd choice. The 1st, 3rd and 4th choices are incorrect as they contain
diamonds instead of squares, and the 5th choice is incorrect as it contains two
triangles but no square.

Therefore, the 2nd choice is the correct answer.

98. Explanation
The correct answer is 2.

The three figures in this question include a bigger shape in the middle with smaller
shapes on top and at the bottom. The smaller shapes are a smaller version of the
upper half of the bigger shape.

This eliminates answer choice 1, as the smaller shapes are different from each other
and from the bigger shape.
Answer choice 5 is also eliminated, as the smaller shapes (pentagons) are the same as
the entire bigger shape (a pentagon), not the upper half.

31
Answer choice 3 is also wrong, as the smaller shapes above and below the bigger
shape show the entire bigger shape, only rotated.

We are now left with answer choices 2 and 4.


If you look closely, you will see that in answer choice 4, the smaller shape at the
bottom is turned upside down, as if it is the lower half of the bigger shape. Since it
must be the upper half, this is incorrect.

The 2nd choice is the only one left that follows these rules, so it is the correct answer.

99. Explanation
The correct answer is 3.

All three figures in this question contain rectangles that are divided into four, with a
circle and a rhombus inside.

This eliminates answer choices 2, 4, and 5, as they contain shapes that are not a circle
and rhombus.

How do we choose between answer choices 1 and 3?


Look again at the three figures in the top row - what else do they have in common?
We can see that in all three figures, the circle and the rhombus appear next to each
other.
The only answer choice which follows this rule is the 3rd choice.
Therefore, the 3rd choice is the correct answer.

100. Explanation

The correct answer is 5.

All three figures in this question contain shapes divided (approximately) into
one-third (1/3) and two-thirds (2/3) parts.
The only answer choice which follows this rule is the 5th choice.
The 1st, 2nd, and 3rd choices are all divided evenly into halves, so they are incorrect.
The 4th choice is clearly not divided in the same way as the shapes in the question, as
the smaller part is much smaller and the larger part is much larger.
Therefore, the 5th choice is the correct answer.

101. Explanation
The correct answer is 4.

In this question all three figures have:


• A rectangle at the bottom,
• A circle in the middle, and –

32
• A triangle at the top.
The only answer choice which follows these rules is the 4th choice.

Therefore, the 4th choice is the correct answer.

102. Explanation
The correct answer is 5.

Each of the three figures in this question has two corners. A corner is where two
straight lines meet.

In answer choice 1 the shape has three such corners.

The shape in answer choice 2 has five such corners.

Answer choice 3 has no corners at which two straight lines meet.


Answer choice 4 has three such corners.

33
Therefore, the 5th choice is the correct answer.

103. Explanation
The correct answer is 5.

Each of the figures in this question has six sides and an oval shape on the inside.

The only answer choice which follows these rules is the 5th choice.

All of the other answer choices contain shapes with fewer than six sides, and the
shapes inside them are not oval.

Therefore, the 5th choice is the correct answer.

104. Explanation
The correct answer is 3.

All three figures in this question contain:


• An outer shape which is a square
• An inner shape whose top part is textured, and
• A background whose bottom part is textured

The only answer choice which follows all of these rules is the third choice.

The first choice is incorrect because the textured parts are the bottom of the inner
shape and the top of the background instead of the other way around.
The second choice is incorrect because the outer shape is a circle instead of a square.
The fourth choice is incorrect because both parts of the inner shape are textured, and
the background has no textured part at all.
The fifth choice is incorrect because there is no inner shape, and the bottom part of
the background has 2 different textures. (an alternative explanation is that the inner
shape is a rectangle and it is fully textured, instead of just the top half.)
Therefore, the 3rd choice is the correct answer.

34
105. Explanation
The correct answer is 2.

All three figures in this question contain:


• An outer shape which is a circle,
• A 5-sided shape (pentagon) inside the circle, and –
• A smaller 4-sided shape (quadrilateral) inside the pentagon

The only answer choice which follows these rules is the 2nd choice.
Therefore, the 2nd choice is the correct answer.

106. Explanation
The correct answer is 5.

In this question all three figures are shapes with three squares, some appearing in 3D,
with one shape in each square.
The squares in all three figures contain a sun, a heart and a banner.

Answer choice 1 is a shaped like a can and does not contain three squares.
Answer choice 2 does contain three squares, but the banner does not look like the one
in the figures above.
Answer choice 3 does not contain three squares with a shape in each one.
Answer choice 4 contains an octagon and not three squares.

Therefore, the 5th choice is the correct answer.

107. Explanation
The correct answer is 1.

Each of the three figures in this question includes:


• Two curly brackets,
• A circle,
• A triangle,
• A hexagon, and –
• A pentagon

This eliminates answer choice 3, as it contains only one curly bracket.


Answer choice 4 can also be eliminated since it contains square brackets and not curly
ones.
Answer choice 2 can be eliminated because it contains two triangles and two circles.
Answer choice 5 can be eliminated because it contains two squares.

We are left with answer choice 1, which is the correct answer.

35
108. Explanation
The correct answer is 3.

In this question, we can see that:


 All three figures consist of an outer shape with a smaller inner shape. The
smaller inner shape is similar to the outer shape.
 All shapes are comprised of straight lines only.
 The outer shape is colored with light blue that fades out from bottom to top.

The 1st choice is incorrect as it is comprised of curved lines.


The 2nd choice is incorrect as it is also comprised of curved lines.
The 4th choice is incorrect as the outer shape is colored with light blue that fades out
from right to left.
The 5th choice is incorrect as the inner shape is colored instead of the outer shape.

Therefore, the 3rd choice is the correct answer.

109. Explanation
The correct answer is 5.

In this question, we can see that:


All three figures are comprised of two shapes.
One of the shapes is located in the front and the other shape in the back.
The shape in the back has one side more than the shape in the front.

We can see that in the 5th answer choice, the shape in the back has six sides, while
the shape in the front has five sides. This means that the shape in the back has one
side more than the shape in the front.

Therefore, the 5th answer choice is the correct answer.

110. Explanation
The correct answer is 3.

In this question, if we straighten the figures in the top row, we can see that all three of
them contain the same shapes, in the same order. The shapes follow this
order (from left to right): a division sign, an arrow, a white triangle, a dark heart, and
a white circle.

The 1st choice is incorrect as, from left to right, the arrow should come before the
triangle and not after it.
The 2nd choice is incorrect as it contains a white heart; the heart should be dark.
The 4th choice is incorrect as, from left to right, the first shape should be the division
sign and not the circle.

36
The 5th choice is incorrect as it contains a dark triangle and a white heart; the triangle
should be white and the heart should be dark.

The 3rd answer choice contains the aforementioned shapes in the exact same order,
from left to right, as the figures in the top row.

Therefore, the 3rd answer choice is the correct answer.

111. Explanation
The correct answer is 1.

In this question, we can see that:


 The three figures are squares divided into nine smaller parts.
 In each figure, there are three Xs and one circle.
 In each figure, the three Xs are organized in a sequence of a straight or a
diagonal line.
 In each figure, the circle is located in one of the bigger square’s corners.

In the 1st answer choice, we can see a square divided into nine smaller parts. This
square contains three Xs and one circle. We can see that the Xs are organized in
a sequence of a straight line and that the circle is located in one of the bigger
square’s corners.

Therefore, the 1st answer choice is the correct answer.

112. Explanation
The correct answer is 3.

In this question, you can see that:

•Each of the three figures consists of two similar shapes, one on top of the other.
•The top shape is similar to the bottom shape, only rotated 90 degrees clockwise.

In the 3rd answer choice, you can see two similar shapes (trapezoids), one on top of
the other. In addition, the top trapezoid is similar to the bottom one, only rotated 90
degrees clockwise.

Therefore, the 3rd answer choice is the correct answer.

The first answer choice shows two shapes side by side instead of one on top of the
other, and therefore it cannot be correct.

The second answer choice has one shape on top, but it has only been rotated through
45 degrees instead of 90 degrees, so it cannot be correct.

37
The fourth answer choice shows two shapes, where the top has been rotated through
180 degrees instead of 90 degrees, so it cannot be correct.

In the fifth answer choice, the top shape has been rotated 90 degrees anticlockwise so
the top arrow is pointing west instead of east. Therefore, it cannot be correct.

113. Explanation
The correct answer is 5.

In this question, we can see that:


 All three figures are shapes divided into smaller parts.
 In each shape, half of the smaller parts are colored: In the circle and the
diamond, four out of eight parts are colored. In the rectangle, two out of
four parts are colored.

In the 5th answer choice, we can see a hexagon divided into six smaller
parts. Three out of six parts are colored, which means that half of the hexagon’s total
area is colored.

Therefore, the 5th answer choice is the correct answer.

114. Explanation
The correct answer is 5.

The first shape is a big five-sided shape (pentagon) with a small six-sided shape
(hexagon) inside. The second shape is a seven-sided shape (heptagon) with a small
eight-sided shape (octagon) inside. The third shape is a big three-sided shape (triangle)
with a small four-sided shape (square) inside.
Therefore, the correct answer will be a big shape with a smaller shape inside that has
one more side than the big shape.
In the 1st answer choice, the small shape has two less sides than the big shape.
In the 2nd answer choice, the big shape does not have any sides.
In the 3rd answer choice, the small shape has three less sides than the big shape.
In the 4th answer choice, the small shape has one less side than the big shape.

115. Explanation
The correct answer is 1.

In this question all of the edges of the shapes in the three figures are straight. The only
answer choice that is like this is the 1st choice. The rest of the choices have one or
more rounded corners.

Therefore, the 1st choice is the correct answer.

38
116. Explanation
The correct answer is 5.

In this question the shapes in all three figures are equilateral (this means that all the
sides of each shape are the same length). The only answer choice that is like this is the
5th choice. The rest of the choices have one or more sides that are a different length
than the rest.

Therefore, the 5th choice is the correct answer.

117. Explanation
The correct answer is 3.

In this question the three figures are all shapes that have a green triangle inside on one
of the sides of the shape. The only answer choice that is like this is the 3rd choice.
The 1st, 2nd and 4th choices are incorrect as the triangle is in a corner of the outer
shape and not on one of the sides. The 5th choice is also incorrect as the triangle it
contains is white and not green.

Therefore, the 3rd choice is the correct answer.

118. Explanation
The correct answer is 1.

In this question all three figures are shapes that are divided by a horizontal and a
vertical line inside. The only answer choice that is like this is the 1st choice. The rest
of the choices are divided by diagonal lines.

Therefore, the 1st choice is the correct answer.

119. Explanation
The correct answer is 3.

In this question the three figures are all four sided shapes filled with a dotted pattern.
The only answer choice that is like this is the 3rd choice. The 1st, 2nd and 5th choices
are incorrect as they are not four sided. The 2nd and 4th choices are also incorrect as
they are filled with a broken line pattern as opposed to a dotted pattern.

Therefore, the 3rd choice is the correct answer.

120. Explanation
The correct answer is 5.

In this question the three figures are all different colored octagons with a narrow plus

39
sign in the middle which is filled with various patterns. Notice that the octagon itself
is always a solid color. The only answer choice that is like this is the 5th choice. The
1st choice is incorrect as it has a wide border around the octagon shape but shouldn't.
The 2nd choice is incorrect because the octagon itself is patterned while the plus sign
isn't and it should be the opposite. The 3rd choice is incorrect as it is a square and not
an octagon. Finally, the 4th choice is incorrect as it has a plus sign in the center that is
too large.

Therefore, the 5th choice is the correct answer.

121. Explanation
The correct answer is (3).

All three figures contain a diagonal line that slants upward toward the right. All the
diagonal lines in the question are parallel, meaning they have the same steepness and
point the same way.
The only answer choice that follows this rule is the third choice. The rest of the
options do not contain a similar diagonal line that is parallel to the other diagonal
lines.

Therefore, the third choice is the correct answer.

122. Explanation
The correct answer is 5.

All three figures in this question contain circles with a vertical line that runs through
the middle.
In each half of the circle, we see a smaller shape with a diagonal line that
slants upwards and towards the middle of the outer circle.

The only answer choice which follows this rule is the 5th choice.
Therefore, the 5th choice is the correct answer.

123. Explanation
The correct answer is 1.

All three figures in this question contain shapes divided in half.


The only answer choice which follows this rule is the 1st choice.
Therefore, the 1st choice is the correct answer.

124. Explanation
The correct answer is 5.

At first glance, we see that all three figures in this question are shapes that point

40
upwards and towards the right.
However, there is more than one answer choice that points upwards and towards the
right.
So, which answer is correct?

Let’s look again at the shapes in the top row. What else do they have in common?
When we look closely, we see that all 3 shapes are comprised of straight lines.
The only answer choice which follows this rule is the 5th choice. Since there are
multiple options that point up and to the right, and answer 5 is the only one of the
choices that is composed of only straight lines, we know that option 5 must be the
right answer even though it points to the left. Pointing to the right is not a relevant
factor since it is sufficient that the three shapes above and option 5 are all composed
of only straight lines.

Therefore, the 5th choice is the correct answer.

125. Explanation
The correct answer is 3.

In this question, we can see that:


• All three figures include a cylinder, a rectangle and a triangle
• The shapes are either white, dark or light blue
• Each shape and shading appear only once in each figure

Answer choice 1 can be eliminated because it contains a hexagon instead of a


rectangle.
Answer choice 2 can be eliminated because more than one shape is dark.
Answer choice 4 can be eliminated because more than one shape is white.
Answer choice 5 can be eliminated as well since the rectangle has been replaced with
a different shape.

The only answer choice which follows all the rules is the 3rd choice.

Therefore, the 3rd choice is the correct answer.

126. Explanation
The correct answer is 4.

All three figures in this question contain the same shape only rotated: a circle
divided into three sections with a smaller shape in each section.
Therefore, we need to find an answer choice which also contains the same shape, no
matter how it is positioned.

41
Answer choices 1, 2 and 3 can be eliminated as they do not contain a circle in the
middle section like the figures in the top row.
When we look more closely at answer choice 5, we see that the square and the
triangle have switched places.

Therefore, we are left with the 4th choice, which is the only correct answer.

127. Explanation
The correct answer is 2.

Each of the three figures in this question includes one curved line and
several straight lines.

In answer choice 1 the shape has two curved lines and not one.
The shape in answer choice 3 has two curved lines as well.
The shape in answer choice 4 has no curved lines at all.
The shape in answer choice 5 has only a curved line and no straight lines.

Answer choice 2 is the only one containing straight lines and one curved line.

Therefore, the 2nd choice is the correct answer.

128. Explanation
The correct answer is 1.

Each of the three figures in this question contains only one dark triangle and only
one white triangle that are opposite each other.
The only answer choice that follows this rule is the 1st choice.
The 2nd and 3rd answer choices include two dark triangles and no white triangles.
The 4th answer choice does not include a dark triangle.
The 5th answer choice does not include a white triangle.

Therefore, the 1st choice is the correct answer.

129. Explanation
The correct answer is 4.

All three figures contain a house with:


• A tiled roof,
• One door and –
• Two rectangular windows.

The only answer choice which follows this rule is the 4th choice.

42
Answer choice 1 has three doors and one window.
Answer choice 2 has only one window and a roof which is not tiled.
Answer choice 3 also has a roof which is not tiled.
Answer choice 5 has circular windows.

Therefore, the 4th choice is the correct answer.

130. Explanation
The correct answer is 5.

All three figures in this question contain a frame with two suns and five clouds.
The only answer choice which follows this rule is the 5th choice.
Answer choice 1 is incorrect because it contains three suns and three clouds.
Answer choice 2 is incorrect because it contains one sun and five clouds.
Answer choice 3 is incorrect because it contains two suns and six clouds.
Answer choice 4 is incorrect because it contains three suns and five clouds.

Therefore, the 5th choice is the correct answer.

131. Explanation
The correct answer is 2.

In this question, we can see that:


 All three figures consist of shapes divided into four parts.
 The symbols shared by each of the three figures are the smiley face and
the sun. These symbols are located in opposite places inside the figures.

The 2nd answer choice is the only one containing a smiley face and a sun located in
opposite places.

Therefore, the 2nd choice is the correct answer.

132. Explanation
The correct answer is 4.

In this question, we can see the following:


All three figures are shapes containing one diagonal inner line.
The inner line divides each shape into two unequal parts.
All three shapes contain a cloud, which is located above the inner line.

In the 4th answer choice, there is a shape with a diagonal inner line, dividing that
shape into two unequal parts. Moreover, this shape contains
a cloud located above the inner line.

43
The 1st and 2nd answer choices are incorrect since in both cases the inner line is not
diagonal and divides the shape into two equal parts.
The 3rd answer choice is incorrect as the cloud is located below the inner line while it
should be above the line.
The 5th answer choice is incorrect as it contains two inner lines instead of one.

Therefore, the 4th answer choice is the correct answer.

133. Explanation
The correct answer is 2.

In this question, we can see that:


 All three figures contain a shape with two inner lines that meet at a single
point.
 One of the inner lines is straight, while the other is curved.

The 1st answer choice is incorrect as the two inner lines are straight.
The 3rd answer choice is incorrect as this shape contains three inner lines.
The 4th answer choice is incorrect as the two inner lines inside the shape do not meet
at any point.
The 5th answer choice is incorrect as the two inner lines are curved.

In the 2nd answer choice, we can see a shape containing two inner lines that meet at
a single point. One of the lines is straight and the other is curved.

Therefore, the 2nd answer choice is the correct answer.

134. Explanation
The correct answer is 5.

In this question, we can see that all three figures in the top row consist of three
identical shapes that share a common area:

44
Similarly, in the 5th answer choice, we can see that all three circles share a common
area:

The rest of the answer choices do not contain an area that is shared by all three shapes.

Therefore, the 5th answer choice is the correct answer.

135. Explanation
The correct answer is 2.

In this question, we can see that:


 All three figures contain a shape with an inner arrow pointing to one of the
shape’s corners.
 In each of the figures, one of the shapes (either the inner arrow or the outer
shape) is white, while the other shape is dark blue.

In the 2nd answer choice, we can see a shape (triangle) with an inner arrow pointing
to one of its corners. In addition, the triangle is dark blue, while the arrow inside it
is white.

Therefore, the 2nd answer choice is the correct answer.

136. Explanation
The correct answer is 4.

In this question the three figures are all shapes that have perfect squares in the center.
The only answer choice that is like this is the 4th choice. The 2nd, 3rd and 5th choices
are incorrect as they contain inner shapes that are not square and have the wrong
number of sides. The 2nd choice is also incorrect as it contains a four sided shape that
is not a square but a diamond.

Therefore, the 4th choice is the correct answer.

45
137. Explanation
The correct answer is 4.

In this question all three figures are shapes with inverted corners (some of the shape's
corners point into the shape). The only answer choice that is like this is the 4th choice.
The rest of the choices only contain corners that point outward.

Therefore, the 4th choice is the correct answer.

138. Explanation
The correct answer is 1.

In this question the three figures all contain a shape with curved sides inside a shape
that only has straight sides. Notice that the inner and outer shapes are always different
colors. The only answer choice that is like this is the 1st choice. The 2nd choice is
incorrect because the inner shape is straight and outer shape is curved while the
opposite should be true. The 3rd choice is incorrect as both the inner and outer shapes
are the same color. The 4th choice is incorrect as both of the shapes have curved sides.
Finally, the 5th choice is incorrect as the both of the shapes have only straight sides.

Therefore, the 1st choice is the correct answer.

139. Explanation
The correct answer is 5.

In this question all three figures are shapes that have no inverted corners (all of the
shapes corners point outward). The only answer choice that is like this is the 5th
choice. The rest of the choices contain corners that point inverted corners (that
point into the shape).
Alternatively, another pattern this question presents is a series of shapes in which each
shape has one side less than the one before. The first shape is a green hexagon (6
sides), followed by a purple pentagon (5 sides) and an orange quadrilateral (3 sides).
The next shape in the series, therefore, would be a triangle (3 sides).
Therefore, the 5th choice is the correct answer.

140. Explanation
The correct answer is 2.

In this question the three figures are all purple octagons with a narrow plus sign in the
middle which is filled with various patterns. Notice that the octagon itself is always
solid purple. The only answer choice that is like this is the 2nd choice. The 1st choice
is incorrect as it has a plus sign in the center that is too large. The 3rd choice is
incorrect as it is a square and not an octagon. The 4th choice is incorrect because the
octagon itself is patterned while the plus sign isn't and it should be the opposite.

46
Finally, the 5th choice is incorrect as it has a wide border around the octagon shape
but shouldn't.

Therefore, the 2nd choice is the correct answer.

141. Explanation
The correct answer is 4.

In this question all three figures are orange circles each of which contains the
following three shapes: a yellow diamond, triangle and circle. The only answer choice
that is like this is the 4th choice. The 1st choice is incorrect as it contains two
diamonds and no triangle. The 2nd and 3rd choices are incorrect as they contain
trapezoids instead of triangles. Finally, the 5th choice is incorrect as it contains four
shapes instead of three (a trapezoid in addition to all of the other shapes).

Therefore, the 4th choice is the correct answer.

142. Explanation
The correct answer is 2.

In this question all three figures are triangles each of which contains the following
three shapes: a tilted square, a triangle and a circle. The only answer choice that is like
this is the 2nd choice. The 1st, 3rd and 4th choices are incorrect as they contain
diamonds instead of squares, and the 5th choice is incorrect as it contains two
triangles but no square.

Therefore, the 2nd choice is the correct answer.

143. Explanation
The correct answer is 2.

The three figures in this question include a bigger shape in the middle with smaller
shapes on top and at the bottom. The smaller shapes are a smaller version of the
upper half of the bigger shape.

This eliminates answer choice 1, as the smaller shapes are different from each other
and from the bigger shape.
Answer choice 5 is also eliminated, as the smaller shapes (pentagons) are the same as
the entire bigger shape (a pentagon), not the upper half.
Answer choice 3 is also wrong, as the smaller shapes above and below the bigger
shape show the entire bigger shape, only rotated.

We are now left with answer choices 2 and 4.

47
If you look closely, you will see that in answer choice 4, the smaller shape at the
bottom is turned upside down, as if it is the lower half of the bigger shape. Since it
must be the upper half, this is incorrect.

The 2nd choice is the only one left that follows these rules, so it is the correct answer.

144. Explanation
The correct answer is 3.

All three figures in this question contain rectangles that are divided into four, with a
circle and a rhombus inside.

This eliminates answer choices 2, 4, and 5, as they contain shapes that are not a circle
and rhombus.

How do we choose between answer choices 1 and 3?


Look again at the three figures in the top row - what else do they have in common?
We can see that in all three figures, the circle and the rhombus appear next to each
other.
The only answer choice which follows this rule is the 3rd choice.
Therefore, the 3rd choice is the correct answer.

145. Explanation

The correct answer is 5.

All three figures in this question contain shapes divided (approximately) into
one-third (1/3) and two-thirds (2/3) parts.
The only answer choice which follows this rule is the 5th choice.
The 1st, 2nd, and 3rd choices are all divided evenly into halves, so they are incorrect.
The 4th choice is clearly not divided in the same way as the shapes in the question, as
the smaller part is much smaller and the larger part is much larger.
Therefore, the 5th choice is the correct answer.

146. Explanation
The correct answer is 1.

In this question, we can see that:


• All three figures include a circle, a square and a rhombus
• The shapes are either white, dark or striped
• Each shape and shading appear only once in each figure

In answer choice 2 we have a pentagon instead of a circle, and in answer choice 3 we


have a heart instead of a rhombus, so they can be eliminated.

48
In answer choices 4 and 5 either the shading or the shape repeat, so they can
be eliminated as well.

Therefore, the 1st choice is the correct answer.

147. Explanation
The correct answer is 4.

In this question all three figures have:


• A rectangle at the bottom,
• A circle in the middle, and –
• A triangle at the top.
The only answer choice which follows these rules is the 4th choice.

Therefore, the 4th choice is the correct answer.

148. Explanation
The correct answer is 5.

Each of the three figures in this question has two corners. A corner is where two
straight lines meet.

In answer choice 1 the shape has three such corners.

The shape in answer choice 2 has five such corners.

Answer choice 3 has no corners at which two straight lines meet.


Answer choice 4 has three such corners.

49
Therefore, the 5th choice is the correct answer.

149. Explanation
The correct answer is 5.

Each of the figures in this question has six sides and an oval shape on the inside.

The only answer choice which follows these rules is the 5th choice.

All of the other answer choices contain shapes with fewer than six sides, and the
shapes inside them are not oval.

Therefore, the 5th choice is the correct answer.

150. Explanation
The correct answer is 3.

All three figures in this question contain:


• An outer shape which is a square
• An inner shape whose top part is textured, and
• A background whose bottom part is textured

The only answer choice which follows all of these rules is the third choice.

The first choice is incorrect because the textured parts are the bottom of the inner
shape and the top of the background instead of the other way around.
The second choice is incorrect because the outer shape is a circle instead of a square.
The fourth choice is incorrect because both parts of the inner shape are textured, and
the background has no textured part at all.
The fifth choice is incorrect because there is no inner shape, and the bottom part of
the background has 2 different textures. (an alternative explanation is that the inner
shape is a rectangle and it is fully textured, instead of just the top half.)
Therefore, the 3rd choice is the correct answer.

151. Explanation

50
The correct answer is 2.

All three figures in this question contain:


• An outer shape which is a circle,
• A 5-sided shape (pentagon) inside the circle, and –
• A smaller 4-sided shape (quadrilateral) inside the pentagon

The only answer choice which follows these rules is the 2nd choice.
Therefore, the 2nd choice is the correct answer.

152. Explanation
The correct answer is 5.

In this question all three figures are shapes with three squares, some appearing in 3D,
with one shape in each square.
The squares in all three figures contain a sun, a heart and a banner.

Answer choice 1 is a shaped like a can and does not contain three squares.
Answer choice 2 does contain three squares, but the banner does not look like the one
in the figures above.
Answer choice 3 does not contain three squares with a shape in each one.
Answer choice 4 contains an octagon and not three squares.

Therefore, the 5th choice is the correct answer.

153. Explanation
The correct answer is 5.

In this question, we can see that:


All three figures are comprised of two shapes.
One of the shapes is located in the front and the other shape in the back.
The shape in the back has one side more than the shape in the front.

We can see that in the 5th answer choice, the shape in the back has six sides, while
the shape in the front has five sides. This means that the shape in the back has one
side more than the shape in the front.

Therefore, the 5th answer choice is the correct answer.

154. Explanation
The correct answer is 1.

In this question, we can see that:

51
 The three figures are squares divided into nine smaller parts.
 In each figure, there are three Xs and one circle.
 In each figure, the three Xs are organized in a sequence of a straight or a
diagonal line.
 In each figure, the circle is located in one of the bigger square’s corners.
In the 1st answer choice, we can see a square divided into nine smaller parts. This
square contains three Xs and one circle. We can see that the Xs are organized in
a sequence of a straight line and that the circle is located in one of the bigger
square’s corners.
Therefore, the 1st answer choice is the correct answer.

155. Explanation
The correct answer is 3.

In this question, you can see that:

•Each of the three figures consists of two similar shapes, one on top of the other.
•The top shape is similar to the bottom shape, only rotated 90 degrees clockwise.

In the 3rd answer choice, you can see two similar shapes (trapezoids), one on top of
the other. In addition, the top trapezoid is similar to the bottom one, only rotated 90
degrees clockwise.

Therefore, the 3rd answer choice is the correct answer.

The first answer choice shows two shapes side by side instead of one on top of the
other, and therefore it cannot be correct.

The second answer choice has one shape on top, but it has only been rotated through
45 degrees instead of 90 degrees, so it cannot be correct.

The fourth answer choice shows two shapes, where the top has been rotated through
180 degrees instead of 90 degrees, so it cannot be correct.

In the fifth answer choice, the top shape has been rotated 90 degrees anticlockwise so
the top arrow is pointing west instead of east. Therefore, it cannot be correct.

156. Explanation
The correct answer is 5.

In this question, we can see that:


 All three figures are shapes divided into smaller parts.

52
 In each shape, half of the smaller parts are colored: In the circle and the
diamond, four out of eight parts are colored. In the rectangle, two out of
four parts are colored.

In the 5th answer choice, we can see a hexagon divided into six smaller
parts. Three out of six parts are colored, which means that half of the hexagon’s total
area is colored.

Therefore, the 5th answer choice is the correct answer.

157. Explanation
The correct answer is 3.
All three figures in this question are divided into two equal halves. In addition, the
separating line is a diagonal line slanted up to the left.
The third figure is divided into two equal halves, and the separating line is a diagonal
line which slants up to the left. Thus, this is the correct answer.
The first answer choice is incorrect as the separating line slants up to the right.
The second answer choice is incorrect as the separating line is horizontal when it
should be diagonal.
The fourth answer choice is incorrect as the separating line slants up to the right.
The fifth answer choice is incorrect as the figure is not divided into two equal halves.

158. Explanation
The correct answer is 1.

All three figures in this question are comprised of:

 An outer shape made up of curved lines only.


 Two identical black inner shapes made up of straight lines only.

The only answer choice which follows these rules is the 1st one. The outer shape is
made up of curved lines only, and the two black inner shapes (hexagons) are made
up of straight lines only.

Therefore, the 1st choice is the correct answer.

159. Explanation
The correct answer is 2.

In this question, all three figures above consist of a big shape in the middle, with
the same two smaller shapes, one on the right and one on the left. In addition, notice
that all the figures are aligned, and that the smaller shapes are in the same direction as
the big shape.

53
The second figure consists of a big pentagon in the middle with two smaller
pentagons on its right and left. Notice that the small pentagons are in the same
direction as the big pentagon, and that the whole figure is aligned. Thus, this is the
correct answer.

The first answer choice is incorrect as the two smaller rectangles are located above
and beneath the big rectangle, instead of being located on its sides.

The third answer choice is incorrect as the small pentagons are pointing downwards
while the big pentagon is pointing upwards, when instead they all should be on the
same direction.

The fourth answer choice is incorrect as the figure is not aligned but rather slants to
the right.

The fifth answer choice is incorrect as the smaller shapes (rectangles) are not the same
as the big shape (the pentagon).

160. Explanation
The correct answer is 1.

In this question, you can see that the three figures above consist of three shapes each,
which are set in a diagonal manner: a circle, a trapezoid, and a star. In addition, in
each figure, the shapes (the circle, the trapezoid, and the star) are either dark
blue, white, or checkered. Notice that there is no match between a specific shape and
a specific color, and that no two shapes in the same figure have the same filling.

The first figure consists of the three correct shapes: the circle, the trapezoid, and the
star. These shapes are correctly set diagonally. In addition, these shapes have the
correct colors: white (the trapezoid), dark blue (the circle), and checkered (the star).
Thus, this is the correct answer.

The second and third answer choices are incorrect as although they consist of the
correct shapes and have the correct colors, the shapes in them are not set diagonally,
and therefore they can be eliminated.

The fourth and fifth answer choices are incorrect as although they have the correct
shapes which are correctly set diagonally, they do not have the correct colors: the
fourth answer choice is not white, and the fifth answer choice is not checkered.
Therefore, these answer choices can be eliminated.

161. Explanation
The correct answer is 3.

54
In this question, the three figures above have straight sides and are filled in
with diagonal lines that slant up to the right.

The third answer choice follows these conditions: it is a shape that consists of only
straight sides, and is filled in with diagonal lines slanted up to the right. Therefore,
this is the correct answer.

The first, fourth, and fifth answer choices are incorrect as they all consist of curved
lines as well as straight lines.
The second answer choice is incorrect as although it consists of straight lines only, it
is filled in with diagonal lines slanted up to the left instead of being slanted up to the
right.

Tip: If you find it difficult to infer the rule connecting the three figures above, it
might be useful to look at the answer choices to see if there are any unusual figures. In
this question, you can see that only answer choices 2 and 3 have straight lines, so you
can check if this is the correct rule.

162. Explanation
The correct answer is 4.

In this question, the three figures above all contain a diagonal line that connects one
of the shape’s corners with one of the shape’s sides. Notice that the line does
not connect two corners.

The shape in the fourth answer choice contains a diagonal line that stretches from its
bottom left corner to the middle of its upper right side. Therefore, this is the correct
answer.

The first answer choice is incorrect as the line in this shape connects two sides and
does not reach a corner.
The second answer choice is incorrect as the line in this shape connects only corners
and does not reach one of the shape’s sides.
The third answer choice is incorrect as there are two lines in this shape while there
should be only one.
The fifth answer choice is incorrect as the line in this shape connects two sides and
does not reach a corner.

163. Explanation
The correct answer is 3.

In this question, the three figures above all contain two inner lines
that intersect inside the shape. Notice that the inner lines divide each figure
into four parts (not necessarily equal).

55
The third answer choice is a pentagon which contains two inner lines that intersect
inside it. In addition, these lines divide the pentagon into four parts. Therefore, this is
the correct answer.

The first and the second answer choices are incorrect as the two inner lines in these
figures do not intersect.
The fourth answer choice is incorrect as this figure contains three inner lines instead
of two.
The fifth answer choice is incorrect as although this figure is divided into four parts, it
contains three inner lines instead of two.

164. Explanation
The correct answer is 1.

In this question, the three figures above are divided into two parts: a white part and
a checkered part. In addition, within the white part, there is a small white
shape which is the same as the outer figure.

The first answer choice is a pentagon divided into a white part and a checkered part.
Within the white part there is a small pentagon, which is the same as the outer figure.
Therefore, this is the correct answer.

The second answer choice is incorrect as the small shape is the same as the shape of
the white part (a trapezoid), while it should be the same as the outer figure (a
hexagon).
The third answer choice is incorrect as the small shape is located within the checkered
part instead of the white part.
The fourth answer choice is incorrect as the small shape (a triangle) is checkered
instead of being white. In addition, the outer figure (except for the inner small triangle)
is white, while it should be both white and checkered.
The fifth answer choice is incorrect as the small shape (a triangle) is different than the
outer figure (a parallelogram).

165. Explanation
The correct answer is 5.

In this question, we can see that:


 The three figures are circles containing two inner shapes, one to each side of
an inner line that cuts the circles in half.
 One of the inner shapes is white, while the other inner shape is black.
 In each figure, the two inner shapes have a different number of sides.

The 1st and 3rd answer choices are incorrect as the two inner shapes in these choices
have the same number of sides.

56
The 2nd and 4th answer choices are incorrect as the two inner shapes in these choices
are the same color.
The 5th answer choice is a circle containing one black shape and one white shape. In
addition, these two shapes have a different number of sides (pentagon with five
sides and triangle with three sides).

Therefore, the 5th answer choice is the correct answer.

57
2. Figure Matrices
1. Explanation
The correct answer is 1.

In the top row we have two figures - how do they go together?


We can see that the two figures look similar, but they some changes have occurred
from the left side to the right:
• The outer shape has received one extra side, making it into a new shape
• The middle shape's rounded side has become straight sides with rounded
corners
• The inner shape has been flipped on its side
• All the shapes have been closed (the missing side has been added without further
changing the shape)

In the bottom row, we have a single figure, and we need to find a figure that will go
together with it in the same way that the two figures above go together.

The only three choices that change the outer shape to one with one more side are 1, 4,
and 5.
Answer choice 1 makes all four changes listed above.
Answer choice 4 does not close the shapes.
Answer choice 5 does not flip the inner shape on its side.

Therefore, the 1st answer choice is the correct answer.

2. Explanation
The correct answer is 2.

In the top row we have two figures - how do they go together?


We can see that the outer shapes remain the same in both figures - a square - but the
inner shapes and shading changes:
• The middle parallelogram on the left is flipped horizontally on the right,
• The dark shading becomes white,
• The checkered shading becomes dark, and -
• The white shading becomes striped.

In the bottom row we have a single figure, and we need to find a figure that will go
together with it in the same way that the two figures above go together.

We can immediately eliminate answer choice 3 because it belongs to the family of


figures in the top row and not the bottom row.
We can also eliminate answer choice 1, because it includes a circle and not
a parallelogram.

58
Answer choice 4 can be eliminated because the star hasn’t remained in place.
Answer choice 5 can be eliminated because the parallelogram has not been flipped.

We are left with the 2nd choice, which is the only correct answer.

3. Explanation
The correct answer is 1.

On the top row, there are two figures - how do they go together?
We can see that the biggest and smallest shapes remain the same in both figures, but
the middle shape and the shading has changed:
The middle circle becomes a square on the right,
The white shading becomes dark,
The dark shading becomes white, and
The grey color fades out from bottom to top instead of from top to bottom.

On the bottom row, there is a single figure. We must determine which figure will go
together with it in the same way the two figures on the top row go together.
We can immediately eliminate answer choice 2. It belongs to the same family as the
figures on the upper row, not the bottom row.

We can also eliminate answer choice 5. It shows checked shading which does not
appear in any of the other figures.

Answer choice 3 can be eliminated because the smallest shape has changed from a
square into a rhombus, while it should have remained a square.
Answer choice 4 can be eliminated because the middle circle hasn’t become a square.

We are left with figure 1 as the only correct answer.

4. Explanation
The correct answer is 1.

In this question you can see that on the top row:


The left figure consists of five different shapes and all of them are white.
When looking at the right figure you can see that the order of the shapes has been
reversed and the 2nd and 4th shapes changed their color to black.

Therefore, on the bottom row the order of the shapes in the right figure should be
(from left to right): lightning, ring-shaped circle, sun, triangle, and trapezoid.
Additionally, the ring-shaped circle and the triangle should be black as they are in the
second and fourth places.
The only answer that meets this criteria is answer 1.

59
Answer 2 is incorrect because the order of the shapes has not been reversed.
Answer 3 is incorrect because the 3rd and 5th shapes are black instead of the 2nd and
4th shapes.
Answer 4 is incorrect because the order of the shapes is incorrect. Tip: this answer
choice can be ruled out right away by noticing that the first shape on the left is a
triangle when it should be lightning.
Answer 5 is incorrect because the 1st, 3rd and 5th shapes are black instead of the 2nd
and 4th shapes.

5. Explanation
The correct answer is 5.

In the top row, from left to right, we can see that:


• The inner shape (a triangle) becomes the outer shape and remains white.
• The outer shape (a rectangle) is doubled into one big rectangle and one smaller
rectangle. These rectangles become the inner shapes. The filling of the rectangles
changes from diagonal lines slanted up to the right into diagonal lines slanted up to
the left. In addition, the smaller rectangle is located on top of the bigger one.

The correct answer must follow the above conditions: The inner shape (a trapezoid)
should become the outer shape and remain white. The outer shape (a rhombus) should
be doubled into one big rhombus and a smaller one. The smaller rhombus should be
placed on top of the bigger one. In addition, the filling of the rhombuses should
change into diagonal lines that are slanted up to the left.

Answer (1) is incorrect as the diagonal lines are slanted up to the right while they
should be slanted up to the left.
Answers (2) and (3) are incorrect as their outer shape should be the trapezoid and not
the rhombus.
Answer (4) is incorrect as the bigger rhombus is placed on top of the smaller rhombus
while the small rhombus should be placed on top of the bigger one.

6. Explanation
The correct answer is 3.

In the top row we have two figures - how do they go together?


We can see that the shapes themselves remain the same in both figures, but the
shading changes:
• The dark shading becomes white,
• The brick texture becomes dark, and -
• The white becomes brick-textured.

In the bottom row, we have a single figure, and we need to find a figure that will go
together with it in the same way that the two figures above go together.

60
We can immediately eliminate answer choice 2 because it belongs to the same family
as the figures in the upper row, and not the bottom row.
We can also eliminate answer choice 5 because the inner shape has changed.
When we look at the shading of the other 3 answer choices, the only one that follows
the same pattern as the figures above is answer choice 3.

Therefore, answer choice 3 is the correct answer.

7. Explanation
The correct answer is 5.

In the top row we have two figures - how do they go together?


We can see that the two figures look similar, but:
• The figure on the left is flipped twice on the right: both horizontally and
vertically.
• The striped texture on the left becomes white on the right, while.
• The dark and checked shading remain the same.

In the bottom row we have a single figure, and we need to find a figure that will go
together with it in the same way that the two figures above go together.

We can immediately eliminate answer choice 1 because it belongs to the family of


figures in the top row, and not the bottom row.
We can also eliminate answer choice 2, because it is positioned in the same way as
the first figure, and is not flipped.
When we look at the other three answer choices, the only one that matches the pattern
is answer choice 5.
Answer choice 3 is flipped only vertically and not horizontally.
In answer choice 4 the striped texture does not become white.

Therefore the 5th answer choice is the correct answer.

8. Explanation
The correct answer is 3.

In the top row, from left to right, the background flips (the frames mirror each
other), half of the shape (the heart) in the foreground (in front) disappears and the
shape itself swaps colors (from white to grey).

In the bottom row we should have the same relationship. We can eliminate the 1st
answer choice because the star did not swap colors, and the 2nd choice because the
star remained whole. We can also eliminate the 4th choice because the background
did not flip, and the 5th choice because the background rotated instead of flipping (the

61
part shaded black is in the lower corner but should be in the upper corner).
We are left with the 3rd choice as the only correct answer.

9. Explanation
The correct answer is 2.

In the top row, the outer shape in the left frame becomes the inner shape in the
right frame, while the inner shape in the left frame becomes the outer shape in the
right frame. The middle shape remains the same. Furthermore all the shapes swap
colors (from black to grey or from grey to black).

In the bottom row we should have the same relationship, as the outer shape in the left
frame is a hexagon, the inner shape in the right frame should also be a hexagon, we
can eliminate the 4th answer choice as the inner-most shape it contains is a pentagon
and not a hexagon. We can also eliminate the 3rd answer choice as it is missing the
middle shape. Finally we can eliminate the 1st and 5th choices because not all the
shapes swapped colors.
We are left with the 2nd choice as the only correct answer.

10. Explanation
The correct answer is 4.

In this question you can see that on the top row:


• There are two shapes that are partly overlapping.
• The figures are the same shape and size.
• In the figure on the left, the top shape is black and is behind the bottom shape
which is white. In the figure on the right, the top black shape moved in front of the
bottom white shape.

Therefore, on the bottom row, the top shape of the right figure should stay black and
move in front of the bottom shape, which should stay white.
The only answer that meets this criteria is answer 4.

Answer 1 is incorrect because the bottom shape is shortened and covers less of the top
shape.
Answer 2 is incorrect because the top shape is white and the bottom shape is black.
Answer 3 is incorrect because both shapes are white.
Answer 5 is incorrect because both shapes are black.

11. Explanation
The correct answer is 2.

Look closely at the top row: As you move across the row, the star is doubled and
is partially overlapping. The parts which overlap become black.

62
Apply the same rule to the bottom row. As you double and partially overlap the
frames, only segments of the top and bottom of the frames will overlap and therefore
become black.
The only answer choice which follows this rule is 2.

12. Explanation
The correct answer is 5.

In the top row we have two figures - how do they go together?


We can see that the shapes themselves remain the same in both figures, but the
shading changes:
• The dark shading on the left becomes white on the right,
• The white becomes striped, and -
• The striped becomes dark.

In the bottom row we have a single figure, and we need to find a figure that will go
together with it in the same way that the two figures above go together.

We can eliminate answer choices 2 and 4 because they are the wrong shape.
When we look at the shading of the other 3 answer choices, the only one that follows
the same pattern as the figures above is answer choice 5.

Therefore, answer choice 5 is the correct answer.

13. Explanation
The correct answer is 5.

In this question you can see that on the top row:


• There are 2 shapes: an inner shape and an outer shape.
• In the left figure the inner shape is white and the outer shape is black.
• When looking at the right figure you can see that the shapes changed their colors–
the inner shape changes from white to black and the outer shape changes from black
to white.
• In the right figure the inner shape is shortened and widened.

Therefore, on the bottom row, the outer shape of the right figure should be white, and
the inner shape should be black. Additionally, the inner shape (a trapezoid) should be
shortened and widened.
The only answer that meets this criteria is answer 5.

Answer 1 is incorrect because the inner shape has changed from a trapezoid to a
rectangle. From left to right, the inner shape should stay the same, only with different
proportions.
Answer 2 is incorrect because the inner shape has not been widened, only shortened.

63
Answer 3 is incorrect because the inner shape has neither been shortened nor widened.
Answer 4 is incorrect because the outer shape is black instead of white and the inner
shape is white instead of black.

14. Explanation
The correct answer is 3.

In this question you can see that on the top row:


• The inner shape is extended to the right until it intersects with the outer shape to
create the figure on the right.
• The outer shape stays the same.

Therefore, in the right figure on the bottom row, the outer shape should be a rectangle
with rounded corners, like the outer shape of the left figure, and the inner triangle
should be extended to the right until it intersects with the outer line.
The answer choice that meets this criteria is answer 3.

Answer 1 is incorrect because the triangle is not extended enough for it to intersect
with the outer line.
Answer 2 is incorrect because the triangle’s base is widened and it is not extended to
the right.
Answer 4 is incorrect because the rectangle has sharp corners.
Answer 5 is incorrect because the triangle is moved to the right and not extended to
the right.

15. Explanation
The correct answer is (C).

Identify the image that completes the pattern by examining how the series of shapes
change across the rows and down the columns within the matrix. The direction in
which you examine the shapes depends on where you can most easily visualize the
relationship between the shapes.
When going through the pictures in each row the shapes rotate counter-clockwise, and
when moving down each column (from top to bottom) the shapes rotate clockwise,
while in the bottom row the stars are deleted.
Answer (C) is the correct answer as it fits the pattern. It does not include stars, the
shapes inside are a clockwise rotated version of the box above it, and a
counter-clockwise rotated version of the box to its left.

16. Explanation
The correct answer is 3.

In this question you can see that on the top row:


• The left figure consists of two shapes – a pentagon and a triangle.

64
• The right figure consists of only one shape and it is different from the shapes on
the left – an octagon.
• The common property of the two figures on the left and right is the number of
sides in each figure. In the left figure there are three sides that belong to the triangle
and five sides that belong to the pentagon. Three sides + five sides = eight sides. In
the right figure there are eight sides of an octagon. In other words, adding up the
number of sides of both shapes on the left figure brings you to the number of sides of
the shape on the right.

Therefore, the right figure at the bottom row should have seven sides because the left
figure has three sides that belong to the triangle and four sides that belong to the
square: three sides + four sides = seven sides.
The only answer that meets this criteria is answer 3.

Answer 1 is incorrect because it has five sides.


Answer 2 is incorrect because it has six sides.
Answer 4 is incorrect because it has eight sides.
Answer 5 is incorrect because it has ten sides.

Solving tip:
Note that a prominent difference between the answer choices is the number of sides.
This indicates that the connection between the left figure and the right figure probably
involves number of sides.

17. Explanation
The correct answer is:

In this matrix:
Across a row (from left to right) the gray area of the circle gets smaller in each
successive frame.
Down a column the circle itself gets smaller.

In the third row the left quarter of the circle is gray, and in the third column only the
bottom quarter of the circle is showing.
Therefore the 1st answer choice is the only correct solution.

18. Explanation
The correct answer is 4.

Across the rows, the number of vertical lines increases by one line. For example, in

65
the first row, the first frame contains one vertical line, the second frame contains two
vertical lines, and the third frame contains three vertical lines. We can see the same
pattern in the second and third rows. Therefore, the empty square in the third row will
contain three vertical lines. We can eliminate answer choice 3, as it contains two
vertical lines.

Down the columns, the number of horizontal lines increase by one line. Take a look at
the first column. In the top frame there aren't any horizontal lines, in the middle frame
there is one horizontal line, and in the third frame there are two horizontal lines. The
second and third columns will exhibit the same pattern. In the third column, the first
frame does not contain any horizontal lines, the second frame contains one horizontal
line, so the third frame will contain two horizontal lines. We can eliminate answer
choices 2 and 5 as they do not contain two horizontal lines. Additionally, notice that
the spacing between the horizontal lines is very large. We can eliminate answer
choice 1 and are left with answer choice 4.

19. Explanation
The correct answer is 1.

In this matrix:
Across a row (from left to right), first one shape is removed (from the upper-right
corner), then two shapes are removed (from the top, the remaining shapes are always
in the same position).
Down a column the shape changes and one shape is added in each successive frame.

We can eliminate the 2nd, 3rd, and 4th answer choices because they contain the
wrong number of shapes. We can also eliminate the 5th answer choice because the top
two shapes should be removed, not the bottom two.

Therefore, the 1st answer choice is the only correct solution.

20. Explanation
The correct answer is 1.

Every column contains two shapes. The left column contains a triangle and a circle,
the middle column contains a circle and a square, and the right column contains a
square and a triangle. Down the column, one shape increases in size while another
shape decreases in size. Take a look at the right column. The triangle becomes
progressively smaller as the square becomes progressively bigger. Therefore, the
answer choice will have a small triangle inside a large square. We can eliminate
choice 4, since it is missing a large square.

Additionally, in each row, each shape of the same size is a different color. In the
bottom row, there is a small, white circle and a small, black square. Therefore, the

66
answer choice will contain a small, gray triangle. We can eliminate answer choices 2
and 5. Furthermore, in the bottom row, there is a large, black triangle, and a large gray,
circle. Therefore, the answer choice will contain a large, white square. We can
eliminate answer choice 3 and are left with answer choice 1 as the correct answer.

21. Explanation
The correct answer is 3.

As this question is composed of many elements, focus on each one separately. Across
the rows, notice that the white frame rotates clockwise around the grid. Therefore, in
the bottom row, the answer choice will contain a white frame in the top-right corner of
the grid. We can eliminate answer choices 2 and 5.

Across the rows, the black “L” shape rotates counterclockwise around the grid.
Therefore, in the bottom row, the black “L” shape will be located in the bottom-left
side of the grid. We can eliminate answer choice 4, as its "L" shape is located in the
wrong position. Additionally, across the rows, the black “L” shapes rotate vertically
from frame to frame. Therefore, we can eliminate answer choice 1 and we are left
with answer choice 3 as the correct answer.

22. Explanation
The correct answer is:

In this matrix:
Across a row (from left to right) the black rectangle gets smaller by a third in each
successive frame.
Down a column the square in each frame rotates 90° clockwise.

We can eliminate the 1st, 3rd, and 5th answer choices because the black shape is not
smaller than the shape from the middle column. We can also eliminate the 2nd answer
choice because the square rotated 180° not 90°.

Therefore, we are left with the 4th answer choice as the only correct solution.

23. Explanation
The correct answer is 4.

In every column and row of this matrix, there is one frame with a gray box, one frame
with a white box, and one frame with a black box. The gray box is the combination of
the other two boxes in that row and column (just like grey is a mix between the colors

67
black and white). That means that any circle that appears in a black or white box in
that row or column will appear in the grey box in the same spot and as the same color
circle. If a circle appears in the same spot in both the black and white boxes, and one
of the circles is white and the other is black, that circle will appear as a grey circle in
the grey box.

Since in the third row and in the third column, there is already both a black box and a
gray box, you should look for an answer choice with a white box. Thus, you can
eliminate answer choices (1), (2), and (3).

This leaves us with choices (4) and (5), so let us check to see if either one follows the
rule. You can start with either one and check it in the column or the row. Let's start
with (4) and check it in the row:

24. Explanation
The correct answer is:

This matrix contains rectangles in four shades: white, light gray, dark gray, and
black. Every frame contains a light gray rectangle which is in a different position in
each row or column. Each of the other three colors appears in two out of three of the
frames in a row or column. In the columns, each color, except for the light grey,
always appears in the same position, while in the rows, the rectangles of the same
color are always in different positions.

You can eliminate answer (1) as it does not contain a light gray rectangle. Since in the
third row (and in the third column), there are already two black rectangles, you can
eliminate all the answer choices with black rectangles—(1), (2), and (4). Finally, you
can eliminate answer (5) as its white and dark gray rectangles are in the wrong
positions. The white rectangle should be in the left of the frame since it needs to keep
its place from the right figure in the middle row, and the dark gray rectangle should be
in the middle of the frame since this is its place from the right figure of the top row.
Therefore, the correct answer is (3).

25. Explanation
The correct answer is 3.

In this question you can see that on the top row the figure on the left is doubled to
create the figure on the right.

On the bottom row the figure on the left consists of two triangles – the first is a blue

68
triangle which has its base on the bottom and the second is a white triangle with its
base on the top. Therefore, the right figure should consist of two blue triangles with
their base on the bottom and two white triangles with their base on the top.
The only answer that meets this criteria is answer 3.

Answer choice 1 is incorrect because the bases of the blue triangles are on top and not
on the bottom, and the bases of the white triangles are on the bottom instead of on top.
Answer choice 2 is incorrect because the figure was not doubled. In addition, the base
of the blue triangle is on the top and the base of the white triangle is on the bottom
and it should be just the opposite.
Answer choice 4 is incorrect because all the triangles are white.
Answer choice 5 is incorrect because the doubled figure is separated.

26. Explanation
The correct answer is (E).

Identify the image that completes the pattern by examining how the series of shapes
change across the rows and down the columns within the matrix. The direction in
which you examine the shapes depends on where you can most easily visualize the
relationship between the shapes.
Across each row the white star moves from left to right. Down each column the white
star moves from up to down; in the top row it is positioned above the black stars, in
the middle row it is positioned in between them, and in the bottom row it is positioned
below them. The empty box is on the bottom row; therefore, the white star will be
below the black stars and it is also in on the right column, thus the star will appear on
the right. Therefore, the correct answer is (E).

27. Explanation
The correct answer is 3.

In this question, from left to right, we can see that:


 In the top row, shapes are inserted into the outer shape (triangle) to create the
figure on the right. The number of these inner shapes is the same as
the number of the outer shape’s sides: In the top row, the outer shape
has three sides, and thus three shapes are inserted. In the bottom row, the
outer shape has four sides, and thus four shapes should be inserted.
 In the top row, each of the newly inserted shapes has one side morethan the
outer shape. The outer shape (triangle) has three sides, and each of the inserted
shapes has four sides (squares). In the bottom row, the outer shape (square)
has four sides, and thus each of the inserted shapes should have five sides
(pentagons).
 The inserted shapes are colored with blue.
The 3rd answer choice follows all the above conditions. Therefore, this is the
correct answer.

69
28. Explanation
The correct answer is 1.

In this question you can see that on the top row:


- The left figure is a diamond divided into two triangles – a blue triangle and a white
triangle.
- In the left figure the same diamond shape appears, but it is rotated
90° counterclockwise.

On the bottom row the figure is a square divided to two rectangles – a blue rectangle
and a white rectangle. The blue rectangle is on the bottom of the figure and the white
rectangle is on the top. The figure on the right should look the same as the figure on
the left, but rotated 90° counterclockwise. Therefore, the right figure should be a
square which is blue on the right and white on the left.
The only answer that meets this criteria is answer 1.

Answer 2 is incorrect because it is rotated 45° clockwise.


Answer 3 is incorrect because it is rotated 180°.
Answer 4 is incorrect because it is rotated 90° clockwise.
Answer 5 is incorrect because it is rotated 45° counterclockwise.

29. Explanation
The correct answer is (A).

Identify the image that completes the pattern by examining how the series of shapes
change across the rows and down the columns within the matrix. The direction in
which you examine the shapes depends on where you can most easily visualize the
relationship between the shapes.
Across each row the image is rotated 90-degrees clockwise. Therefore, the empty box
will contain a shape similar to the box on its left, but rotated, making answer (A)
correct.

30. Explanation
The correct answer is 2.

In the top row, from left to right, we added a duplicate shape (another arrow that
was flipped 180 degrees) over the original shape and the overlapping area became
black.

In the bottom row we should have the same relationship. Since in the left frame we
have a square ring shape, in the right frame the shape should be doubled, so we can
eliminate the 1st answer choice. Further, only the overlapping areas should be black,
so we can also eliminate the 3rd, 4th and 5th choices.
We are left with the 2nd answer choice as the only correct answer.

70
31. Explanation
The correct answer is image 2.

In the top row, from left to right:


 The arrow figure rotates 45 degrees clockwise.
 The circled figure on top of it rotates 90 degrees clockwise.
In the bottom row, we should have the same relationship.
We can eliminate images 1, 3 and 5 because the figure underneath the 3\4 circle is not
rotated 45 degrees clockwise.
We can eliminate image 4 because the 3\4 circle is rotated 270 degrees clockwise.
This leaves image 2 - the correct answer.

32. Explanation
In the top row, from left to right:
 The figure has been sliced diagonally and only the top left side of it remains.
 The figure became thicker.
In the bottom row, we should have the same relationship.
Image 1 is incorrect because, even though the top left side remained, the figure
became thinner.
Image 2 and 4 are incorrect because their figures are not sliced diagonally.
Image 5 is incorrect because the top right side of the figure has remained.
Image 3 is the only figure that matches the pattern in the top row, therefore, it is the
correct answer.

33. Explanation
The correct answer is (D).
Identify the image that completes the pattern by examining how the series of shapes
change across the rows and down the columns within the matrix. The direction in
which you examine the shapes depends on where you can most easily visualize the
relationship between the shapes.
Every box contains the same shapes – a square, a triangle and a circle, and one of
those shapes is patterned with diagonal lines. You can notice that each column has the
exact same shape, thus the empty box will contain a shape similar to its column.
Therefore, you can rule out answers (B), (C) and (E).
In addition, when moving across each row the patterned shape is either the inner,
middle or outer one: The top row has the outer shape colored, the colored shape in the
middle row is the one in the middle, and the bottom row has the innermost colored.
As the empty box is in the last row, the patterned shape will be the inner shape.
Therefore, the correct answer is (D).

34. Explanation
The correct answer is 5.

Look carefully at the change in the top row from left to right.

71
The background changes from grey to white, and each circle changes to a
square with the shaded part switching from left to right, or right to left (but not up
and down).

In the bottom row we should have a similar relationship. We can easily eliminate the
2nd answer choice because it contains circles (not squares), and the 4th choice
because the background is grey (not white). We can also eliminate the 1st choice
because it changed in the opposite way than how it was supposed to (the black section
rotated from its position in the left frame and switched sides from its position in the
frame above, while it should have done the opposite). Finally, we can eliminate the
3rd answer choice because the black sections moved in different directions for each
shape, and didn't follow any specific rule.

We are left with the 5th choice as the only correct answer.

35. Explanation
The correct answer is 2.

The relationship between the first two figures is as follows: the shapes in the first
frame each become shapes with one less side, the black shape becomes white and
the white shape becomes black. The correct answer must have the same relationship
with the third figure. The correct answer will be a large black triangle with a small
white four-sided shape.

This means that 2 is the correct answer.

36. Explanation
The correct answer is 2.

There are two rules connecting the two shapes. In the top row, from left to right:
1.The shape on the right has been rotated 180 degrees.
2.The shading has changed on the checked part of the shape. In the shape on the right,
the checked pattern, which was on the left shape, has been replaced by light blue
shading.

You need to look for a shape that follows these two rules for the pyramid shape.
Answer choices 1 and 3 have not been rotated correctly through 180 degrees, and
therefore they can be eliminated.

Using rule number 2, look at shapes 2, 4, and 5. In the original shape on the second
row, the patterned part was in the middle section, so look for a shape with light blue
shading in the middle section. Only shape 2 matches this. Check that the other shaded
parts have stayed the same. The white is still white and the dark blue is still dark blue.
Therefore, 2 is the correct answer.

72
37. Explanation
The correct answer is 3.

In the top row, from left to right:


 The 10-sided black star (the background shape) becomes an 8-sided grey star
(reduced by 2 sides).
 The 4-sided grey rhombus (the foreground shape) becomes a 3-sided black
triangle (reduced by one side).

In the bottom row, we should have the same relationship: The background shape (now
the pentagon) should be reduced by 2 sides (to become a 3-sided shape) and turn grey,
and the foreground shape (now the parallelogram) should be reduced by 1 side (to
become a 3-sided shape) and become black.
The only answer that follows these rules is answer 3, as the background shape is grey
and has 3 sides, and the foreground shape is black and has 3 sides.

Note: By looking at the points in the top row, it seems that the background figure is
reduced by 1 (from a 5 pointed star to a 4 pointed star), so you may have been looking
for the pentagon to also be reduced by 1 (to a 4 sided shape). This is incorrect,
because each point of the star is two sides. If you wanted to argue that the rule should
follow the points of the star instead of the sides (the 5 pointed pentagon should be
reduced to a 4 pointed shape, much like the 5 pointed star becomes a 4 pointed star),
this would work in theory, but there are no answers that would follow all of the rules.
The only answer that presents a 4-pointed grey shape is answer 2, and the foreground
shape inside it is not black, so it cannot be correct. Therefore, you know that you must
find another way of categorizing the shapes other than by the points, such as by the
sides, until you find one correct answer.

Tip:In these types of questions where shapes change to other shapes, always look at
the number of sides and edges first. Even though you are drawn to the points, it is
more likely that the rule will be based on the sides, so it is better to start from there
and see if you can find one valid answer.

38. Explanation
The correct answer is 2.

Look closely at the top row: As you move across the row, the star is doubled and
is partially overlapping. The parts which overlap become black.

Apply the same rule to the bottom row. As you double and partially overlap the
frames, only segments of the top and bottom of the frames will overlap and therefore
become black.

The only answer choice which follows this rule is 2.

73
39. Explanation
The correct answer is 2.

In the top row, from left to right, the figure changes as follows:
• The top shape (circle) becomes the middle shape in the second figure.
• The middle shape (pentagon) becomes the outermost shape in the second figure.
• The bottom shape (arrow) rotates 90° counterclockwise, changes its color to blue,
and becomes the innermost shape in the second figure.
In the bottom row, there should be the same relationship:
•The top shape (circle) should become the middle shape.
•The middle shape (pentagon) should become the outermost shape.
•The bottom shape (moon) should be rotated 90° counterclockwise, colored in
with blue, and become the innermost shape.

The second answer choice follows all the above conditions. Therefore, this is the
correct answer.

The third answer choice is incorrect as the moon inside this figure is rotated 90°
clockwise, instead of 90° counterclockwise.

Alternatively, the bottom shape rotates 90 degrees counterclockwise, changes to blue,


and then moves inside the top shape.
Next, the top shapes move into the middle shape.

40. Explanation
The correct answer is 4.

In this question you can see that on the top row:


- On the left figure there are six triangles. Four of them are white and two are grey.
- On the right figure you can see that the grey triangles have been omitted and the
white triangles have remained in their original position.

Therefore, on the bottom row the figure on the right should consist of two white
squares which are in the same position as the white squares on the left figure, one
square is higher than the other and placed to the left of it. The two grey squares should
be omitted.
The only answer that meets this criteria is answer 4.

Answer 1 is incorrect because the position of the white squares should be reversed.
Answers 2, 3, and 5 are incorrect because they consist of grey squares which should
be omitted.

74
41. Explanation
The correct answer is 1.

In the top row, from left to right, the colors/patterns of the rectangles swap as
follows: dark blue with diagonal stripes (and vice versa), and light blue with a
checked pattern (and vice versa).

In the bottom row we should have the same relationship. In the left frame we have
two striped rectangles on top, so in the right frame they should become dark blue. We
can eliminate the 2nd and 4th answer choices because the upper rectangles did not
change.

The bottom rectangles in the left frame are light blue, so in the right frame they
should have a checked pattern. We can further eliminate the 3rd and 5th choices
because they do not contain a checked pattern on the bottom.

We are left with the 1st choice as the only correct answer.

42. Explanation
The correct answer is 4.

In this question you can see that on the top row:


- The left figure is a blue cross.
- In the right figure there are four blue squares that are placed according to the white
spaces of the cross on the left. Thus, if both the cross on the left and the blue squares
on the right would be combined, they would form a whole square.

In the bottom row the blue cross is rotated. Therefore, the right figure on the bottom
row should consist of four rotated blue squares that are placed according to the white
spaces of the cross on the left. Thus, if both figures on the right and left would be
combined they would form a rotated whole square.
The only answer that meets this criteria is answer 4.

Answer 1 is incorrect because lines were added to the cross to create a square instead
of it being four separated squares that complete the cross to form a square.
Answer 2 is incorrect because it consists of white triangles and not blue squares.
Answer 3 is incorrect because the squares are white instead of blue.
Answer 5 is incorrect because it is a white cross and there are no squares.

43. Explanation
The correct answer is 4.

As we move from left to right across the top row:


The shapes stay in the same location inside the box

75
and remain similar in size,
But -
the triangles change into arrows,
and the white and grey shading is reversed.

The answer must have the same relationship with the left box in the bottom row.

The only answer choice which follows these rules is the 4th.

44. Explanation
The correct answer is 3.

In the top row, the left frame contains a circle, two diagonal lines, and a vertical line.
The right frame contains only the diagonal lines.
The same relationship will hold for the bottom row.
The answer choice will contain only the three diagonal lines present in the bottom-left
frame.

Therefore, answer choice 3 is the correct answer.

45. Explanation
The correct answer is 3.
In the top row, from left to right:
 the grey triangle becomes white;
 the white figure that covers it becomes black and rotates by 90 degrees
clockwise.
In the bottom row, we should have the same relationship:
 A white figure should become black.
 A grey figure should become white.
These rules eliminate figures 2, 4 and 5.
The figure that covers the background figure should rotate by 90 degrees clockwise,
which means the correct answer is 3.

46. Explanation
The correct answer is 2.

In the top row, from left to right, we can see that the figure is rotated 90 degrees
counterclockwise. In addition, two vertical lines are added to the left of the figure’s
center. Notice that these lines do not connect the edges of the figure.

The figures in the bottom row must maintain the same relationship. Answer choice 2
follows these conditions and therefore this is the correct answer.

Answer choice (1) is incorrect as one of the vertical lines connects the edges of the

76
figure.
Answer choice (3) is incorrect as the figure has been rotated only 45 degrees
counterclockwise instead of 90 degrees.
Answer choice (4) is incorrect as the figure has not been rotated.
Answer choice (5) is incorrect as the vertical lines have been added to the right of the
figure’s center while they should be on the left.

47. Explanation
The correct answer is 3.

In the top row we have two figures - how do they go together?


As we move from left to right:
• The shapes change from triangles to circles,
• The number of shapes and their position remain the same, yet -
• All the shapes but one change color from white to black.

In the bottom row we have a single figure, and we need to find a figure that will go
together with it in the same way that the two figures above go together.

In answer choice 5 we have three circles and not two, so it can be eliminated.
In answer choices 1 and 4 none of the circles have changed color, so we can eliminate
them as well.
In answer choice 2 the shapes have changed position, so it is not a correct answer
either.

We are left with answer choice 3, which is the correct answer.

48. Explanation
The correct answer is 5.

Across the rows, compare the colors of the squares in the left column with the colors
of the squares in the right column. The following rules apply:
 White squares change to grey
 Grey squares change to white
 Black squares remain the same color
Therefore, the correct answer choice must be 5.

49. Explanation
The correct answer is 5.

In this question, from left to right, we can see that:


 In the top row, the left-top shape (the smiley face) is flipped over
upside-down but stays in its place. Thus, in the bottom row, the left-top
shape (the heart) should also be flipped over upside-down but stay in its place.

77
 In the top row, the left-bottom shape (the arrow) stays in its place and does
not change at all. Thus, in the bottom row, the left-bottom shape (the
trapezoid) should also stay in its place.
 In the top row, the right-top shape (the “L-shape”) and the right-bottom shape
(the triangle) switch places. Additionally, the new right-top shape (the
triangle) changes its color from white to black. Thus, in the bottom row, the
right-top shape (the star) and the right-bottom shape (the arc) should switch
places, and the new right-top shape (the arc) should change its color from
white to black.

The 5th answer choice follows all the above conditions. Therefore, this is the correct
answer.

50. Explanation
The correct answer is 3.

In the top row we have two figures - how do they go together?


We can see that the two figures look similar, but:
• The figure on the left is flipped horizontally on the right.
• Also, the circle is now dark and has moved behind the shape.

Two new shapes have been added to the figure:


• A vertical line next to the dark circle, and -
• A white circle in the corner above it.

In the bottom row, we have a single figure, and we need to find a figure that will go
together with it in the same way that the two figures above go together.

We can immediately eliminate answer choice 4 because it belongs to the family of


figures in the top row, and not the bottom row.
We can also eliminate answer choice 2, because it is positioned in the same way as
the first shape, and is not flipped.
When we look at the other three answer choices, we see that the only one that matches
the pattern is answer choice 3.
Notice that answer choice 5 includes nearly all the changes we need, but it is a closed
shape and not an open one like the figures in the question.

Therefore the 3rd answer choice is the correct answer.

51. Explanation
The correct answer is 5.

In this question you can see that on the top row:


- The left figure consists of three separate circles: two circles on the top and one on

78
the bottom.
- When looking at the right figure you can see that the circles moved towards each
other to the point that they overlap, and as a result, there is an area that is common to
all three circles.

Therefore, on the bottom row, the right figure should consist of three triangles, one on
top and two at the bottom, and they should partly overlap and have an area that is
common to all three of them.
The only answer that meets this criteria is answer 5.

Answer 1 is incorrect because there is no common area to all three triangles.


Answer 2 is incorrect because there are two triangles on top instead of one, and there
is one triangle on the bottom instead of two.
Answer 3 is incorrect because the triangles do not overlap.
Answer 4 is incorrect because the triangles are not arranged in the right way – there is
a triangle on top, in the middle, and on the bottom, instead of one on top and two at
the bottom.

52. Explanation
The correct answer is 2.

In the top row, there are two figures — how do they go together?
It seems that in the figure on the right, only the bottom triangle stays.
Thus, in the bottom row, let’s look for an answer choice that is the same as the bottom
diamond.

We can see that there is no such answer choice. Therefore, we need to think of
another way the two figures in the top row go together.

If we look closely at the right figure in the top row, we can see that it consists of only
the lines shared by both triangles on the left. Both triangles on the left have two
vertical lines. However, the top triangle has one additional horizontal line which the
bottom triangle doesn't have. Thus, in the figure on the right, this horizontal line
disappears.

In the bottom row, we should have the same relationship. The correct answer should
consist of only the lines shared by both diamonds on the left. Both diamonds on the
left have one horizontal line. However, the bottom diamond has one additional short
diagonal line which the top diamond doesn't have. Thus, in the figure on the right, this
short diagonal line should disappear.

The 2nd answer choice follows this condition. Therefore, the correct answer is 2.

79
53. Explanation
The correct answer is 3.

In the top row we have two figures - how do they go together?


As we move from left to right:
• The white triangles change into black diamonds,
• The hearts rotate 180 degrees but remain gray,
• The number of circles increases by one, and -
• The circles change colors from black to white
In the bottom row, we have a single figure, and we need to find a figure that will go
together with it in the same way that the two figures above go together.

We can eliminate answer choice 2 as it contains a white diamond and not a black one.
In addition, this answer choice contains black circles instead of white ones.
We can also eliminate answer choice 1 as it contains a white heart and not a gray one.
We can eliminate answer choice 4 as it contains three circles instead of four.
Finally, we can eliminate answer choice 5 as it contains a heart that has not been
rotated.

We are left with answer choice 3, which is the correct answer.

54. Explanation
The correct answer is 1.

In the top row, when we move from the left box to the right box, the diagonal
lines are removed and the vertical and horizontal lines remain.

When we look at the answer choices to find the one which will have the same
relationship with the left box in the bottom row, the only answer which follows these
rules is the 1st.

55. Explanation
The correct answer is 3.

In the above matrix (from left to right):


- Across the rows: The shapes rotate 45 degrees (1/8 of a circle).
- Down the columns: The shapes in the third row are the combinations of the shapes
in the first and second rows.

The answer that follows the above patterns is 3.

56. Explanation
The correct answer is 2.

80
In the above matrix (from left to right):
- Across the rows: One black dot is removed (always from the top-right corner)
- Down the columns: One black dot is added (always to the bottom-left corner).

The answer that follows the above patterns is 2.

57. Explanation
The correct answer is 5.

Each frame contains an image composed of an inner square and an outer diamond.
Each row and column a white, gray, and black inner square and a white, gray, and
black outer diamond.
The third row already contains one gray and one black inner square. Therefore, the
inner square of the answer choice must be white. We can eliminate answer choices 1,
2, and 3.
The third row contains one white and one gray outer diamond. Therefore, the outer
square of the answer choice must be black. We can eliminate answer choice 4 and are
left with answer choice 5.

Additionally, notice that the colors of the top-left and bottom-left images switch. The
color of the inner square becomes the color of the outer diamond and the color of the
outer diamond becomes the color of the inner square. This pattern also applies to the
top-right and bottom-right corners. The white, outer diamond will become the color of
the inner square and the inner yellow square will become the color of the outer
diamond in the answer choice.

58. Explanation
The correct answer is 4.

In the above matrix, the pattern goes across the rows as well as down the columns. In
each row or column the number of hearts and their color change from square to square.
Each square in each row or column has a different amount and a different color of
hearts.
In the top row, 2 pink hearts are followed by 3 purple hearts, followed by 1 black
heart. In the first column, 2 pink hearts are followed by 1 purple heart, followed by 3
black hearts. In the bottom row, there are 3 black hearts, followed by 1 pink heart.
Since there are 3 hearts in one square, and 1 heart in the other square, the missing
square must have 2 hearts. Since there are black hearts in the first square and a pink
heart in the second square, the last square must contain purple hearts. Therefore, the
correct answer is answer choice 4, which contains 2 purple hearts. The same pattern
can be seen in the last column as well.

Tip: It is sometimes helpful to see if a pattern appears in the diagonal boxes.

81
59. Explanation
The correct answer is 2.

In this matrix:
Across a row, the rightmost frame is the overlapping gray section of the previous two
frames.
Down a column, the gray part either increases by a ⅛ section in a clockwise manner
or decreases by a ⅛ section in a counter-clockwise manner.

Since in the third column the gray part is decreasing, we can eliminate the 1st, 3rd,
and 5th answer choices, as they all have gray sections that are the same or larger than
the previous (above) frame. We can also eliminate these choices because they do not
contain only the overlapping gray area of the previous two frames in the row. We can
also eliminate the 4th answer choice because the gray section is in the wrong position.

We are left with the 2nd answer choice as the only correct solution.

60. Explanation
The correct answer is:

Across the row, the shapes remain the same.


In the top row, the large form is a circle, the medium form is a triangle, and the small
form is an arrow.
In the middle row, the large shape is a rounded square, the medium shape is a
parallelogram, and the small shape is an arrow.
In the bottom row, the large shape is a pentagon, the medium shape is a triangle, and
the small shape is an arrow.
We can already eliminate answer choice two as it contains an outer, rounded square
rather than a pentagon.
Additionally, across the rows, each large, medium, and small shape is presented in
three colors. Each shape does not appear in the same color twice. Therefore, in the
bottom row, the answer choice will contain a white pentagon, a gray triangle, and a
white arrow. We can further eliminate answer choice three as it has an image with the
wrong colors.
We are left with answer choices 1, 4, and 5. We can further see that the medium
shapes rotate 90 degrees clockwise across each row, and the small shapes (arrows)
rotate 45 degrees counterclockwise. We can also eliminate answer choices 4 and 5
and are left with answer 1.

82
61. Explanation
The correct answer is 5.

This matrix (like many others) is composed of nine squares divided into three rows or
three columns. Each square of the matrix contains a grid of nine squares, also divided
into three rows or three columns, with a circle and a star in each one.
In order to avoid confusion, we will refer to the greater division of squares as
the matrix, and the smaller ones in each square as grids.
The circles and stars appear in different squares in the grid for each square of
the matrix. To find the answer, we can track their movement across the rows of
the matrix, or down the columns of the matrix.
Solving tip: Using the rows is clearly easier in this problem, so it is better to use them
than the columns. However, we will explain them both.
1. Across the rows of the matrix from the left square to the right:
The circle remains in the same row of the grid, and moves one spot to the
right in the grid with each step. The star remains in the same column of
the grid, and moves one spot up in the grid with each step.
2. Down the columns of the matrix from the top square to the
bottom: The circle remains in the same column of the grid, and moves one
spot up in the grid with each step (starting back to the bottom when it reaches
the top).The star remains in the same row of the grid, and moves one spot to
the left in the grid with each step. (In the upper-right frame the star and circle
end up in the same position and the circle covers the star).
Note: As you can tell from the top left square, if the circle and start end up in the
dame square on the grid, the circle covers the star.
Therefore, following either the rows or the columns, the position of the circle in the
bottom left square of the matrix should be the middle-right square of the grid, and the
position of the star should be the top-left square of the grid.

Only the 5th answer choice matches this description, so it is the correct answer.

62. Explanation
The correct answer is 2.

In this matrix:
Across a row and down a column, within every frame, each position has a different
shape and the upper-right corner is always empty.
We can eliminate the 3rd answer choice, because it has an empty spot in the wrong
place.
Since in the third row and column we do not have an O in the lower-left corner, we
should look for an answer choice with an O in the lower-left corner, and we can
eliminate the 4th and 5th choices.
We can eliminate the 1st answer choice, because we already have a frame with an X

83
in the lower-right corner.
Therefore, we are left with the 2nd answer choice as the only correct solution.

63. Explanation
The correct answer is 1.

In the top row we have two figures - how do they go together?


We can see that the two figures look similar, but they some changes have occurred
from the left side to the right:
• The outer shape has received one extra side, making it into a new shape
• The middle shape's rounded side has become straight sides with rounded corners
• The inner shape has been flipped on its side
• All the shapes have been closed (the missing side has been added without further
changing the shape)

In the bottom row, we have a single figure, and we need to find a figure that will go
together with it in the same way that the two figures above go together.

The only three choices that change the outer shape to one with one more side are 1, 4,
and 5.
Answer choice 1 makes all four changes listed above.
Answer choice 4 does not close the shapes.
Answer choice 5 does not flip the inner shape on its side.

Therefore, the 1st answer choice is the correct answer.

64. Explanation
The correct answer is 4.

In the top row we have two figures - how do they go together?

We can see that the two figures look similar, but:


• On the right, the figure is flipped horizontally,
• The long vertical line has been removed,
• The short vertical line now runs through entire length of the shape, and -
• A new horizontal line has been added.

In the bottom row we have a single figure, and we need to find a figure that will go
together with it in the same way that the two figures above go together.

We can immediately eliminate answer choice 3 because it looks like the shape in the
top row, and not the bottom row.
We can also eliminate answer choices 1 and 5, because they have two vertical lines,
while we only need one.

84
When we look at the other two answer choices, the only one that matches the pattern
is answer choice 4.
In answer choice 2 there are two horizontal lines while we need only one.

Therefore the 4th answer choice is the correct answer.

65. Explanation
The correct answer is 2.

The relationship between the first two figures is as follows: the second figure is the
same as the first figure, but rotated 90° counterclockwise, and with a triangle
added on top. The correct answer must have the same relationship with the third
figure.

This means that 2 is the correct answer.

66. Explanation
The correct answer is 2.

Look closely at the top row: As you move across the row, the star is doubled and
is partially overlapping. The parts which overlap become black.

Apply the same rule to the bottom row. As you double and partially overlap the
frames, only segments of the top and bottom of the frames will overlap and therefore
become black.

The only answer choice which follows this rule is 2.

67. Explanation
The correct answer is 4.

Take a look at the above image:


Notice that the shapes that have the same frame correspond with each other and move
in a similar way across the rows:

85
•The red rectangle on the left of the box moves to the right, adjacent to the wall of
the box,
•The blue rectangle on the right of the box moves to the left, and
•The green rectangle moves from behind the blue rectangle to behind the red
rectangle.

The same pattern should occur in the bottom row. The only answer choice that
follows these rules is the 4th.

68. Explanation
The correct answer is 1.

Take a look at the top row. The order of the shapes in the left frame determines the
position of the shapes in the right frames.
 The top shape (the star) is positioned in the center,
 The middle shape (the circle) is positioned on the outside, and -
 The bottom shape (the triangle) is positioned in between the two objects.

The same rule will apply in the bottom row:


The circle sector will be in the center, so we can eliminate answer choices 4 and 5.
The plaque will be the outer shape, so we can eliminate answer choice 2.
The ring will be positioned in between both shapes. We can eliminate answer choice 3,
and are left with answer choice 1 as the correct answer.

69. Explanation
The correct answer is 1.

In the top row, when we move from the left box to the right box, the diagonal
lines are removed and the vertical and horizontal lines remain.

When we look at the answer choices to find the one which will have the same
relationship with the left box in the bottom row, the only answer which follows these
rules is the 1st.

70. Explanation
The correct answer is 2.

In the top row we have two figures - how do they go together?

• The shapes in the right figure overlap in the opposite order than they do in the left
figure: on the left, the lower shape always covers the higher shape, and on the right,
the higher shape always covers the lower shape.

86
• Also, the shape in front on the left and the shape in front on the right are always
the same color, while the other two shapes switch colors with each other.

In the bottom row we have a single figure, and we need to find a figure that will go
together with it in the same way that the two figures above go together.

In answer choices 1, 3 and 5 the shape at the front is not gray, so these choices can be
eliminated.
In answer choice 4 the black and white shapes have remained in place, so we can
eliminate it as well.

We are left with answer choice 2, which is the correct answer.

71. Explanation
The correct answer is 3.

In the top row we have two figures - how do they go together?


As we move from left to right:
• The shapes change from triangles to circles,
• The number of shapes and their position remain the same, yet -
• All the shapes but one change color from white to black.

In the bottom row we have a single figure, and we need to find a figure that will go
together with it in the same way that the two figures above go together.

In answer choice 5 we have three circles and not two, so it can be eliminated.
In answer choices 1 and 4 none of the circles have changed color, so we can eliminate
them as well.
In answer choice 2 the shapes have changed position, so it is not a correct answer
either.

We are left with answer choice 3, which is the correct answer.

72. Explanation
The correct answer is 5.

In the top row we have two figures - how do they go together?


As we move from left to right:
• The shapes on the right mirror the shapes on the left,
• The circle moves in front of the triangle,
• The background color changes from gray to black,

87
• The triangle changes from black to gray, and –
• The circle changes from gray to white.

In the bottom row we have a single figure, and we need to find a figure that will go
together with it in the same way that the two figures above go together.

Answer choice 2 can be eliminated because the background hasn’t changed to black.
Answer choice 3 can be eliminated because the circle remained at the back of the
triangle.
Answer choice 4 can be eliminated because the circle has move higher than in the
figure on the left.
Answer choice 1 can be eliminated because the square has moved to the back of the
triangle while it should have remained in place.

Answer choice 5 is the only one that follows all of the above rules and is the correct
answer.

73. Explanation
The correct answer is 1.

In the top row, from left to right, the figure changes as follows:

 A mirror image of the pentagon is added, forming an upside down pentagon


attached to the original one from below.
 The original pentagon changes its color from white to black.

In the bottom row, we should have the same relationship: A mirror image of the
triangle should be added to form an upside down triangle. This upside down triangle
should be attached to the original triangle from below. In addition, the original
triangle should change its color from white to black. Therefore, the 1st choice is the
correct answer.

The 5th choice is incorrect as the correct relationship between the figures does not
involve flipping the original figure from right to left

74. Explanation
The correct answer is 2.

In the top row, from left to right, the figure changes as follows:

•The top shape (circle) becomes the middle shape in the second figure.
•The middle shape (pentagon) becomes the outermost shape in the second figure.
•The bottom shape (arrow) rotates 90° counterclockwise, changes its color to blue,
and becomes the innermost shape in the second figure.

88
In the bottom row, there should be the same relationship:

•The top shape (circle) should become the middle shape.


•The middle shape (pentagon) should become the outermost shape.
•The bottom shape (moon) should be rotated 90° counterclockwise, colored in
with blue, and become the innermost shape.

The second answer choice follows all the above conditions. Therefore, this is the
correct answer.

The third answer choice is incorrect as the moon inside this figure is rotated 90°
clockwise, instead of 90° counterclockwise.

Alternatively, the bottom shape rotates 90 degrees counterclockwise, changes to blue,


and then moves inside the top shape.
Next, the top shapes move into the middle shape.

75. Explanation
The correct answer is 5.

In this question, from left to right, we can see that:

 In the top row, the left-top shape (the smiley face) is flipped over
upside-down but stays in its place. Thus, in the bottom row, the left-top
shape (the heart) should also be flipped over upside-down but stay in its place.
 In the top row, the left-bottom shape (the arrow) stays in its place and does
not change at all. Thus, in the bottom row, the left-bottom shape (the
trapezoid) should also stay in its place.
 In the top row, the right-top shape (the “L-shape”) and the right-bottom shape
(the triangle) switch places. Additionally, the new right-top shape (the
triangle) changes its color from white to black. Thus, in the bottom row, the
right-top shape (the star) and the right-bottom shape (the arc) should switch
places, and the new right-top shape (the arc) should change its color from
white to black.

The 5th answer choice follows all the above conditions. Therefore, this is the correct
answer.

76. Explanation
The correct answer is 3.

In this question, from left to right, we can see that:

89
 In the top row, shapes are inserted into the outer shape (triangle) to create the
figure on the right. The number of these inner shapes is the same as
the number of the outer shape’s sides: In the top row, the outer shape
has three sides, and thus three shapes are inserted. In the bottom row, the
outer shape has four sides, and thus four shapes should be inserted.
 In the top row, each of the newly inserted shapes has one side more than the
outer shape. The outer shape (triangle) has three sides, and each of the inserted
shapes has four sides (squares). In the bottom row, the outer shape (square)
has four sides, and thus each of the inserted shapes should have five sides
(pentagons).
 The inserted shapes are colored with blue.

The 3rd answer choice follows all the above conditions. Therefore, this is the correct
answer.

77. Explanation
The correct answer is (A).
Identify the image that completes the pattern by examining how the series of shapes
change across the rows and down the columns within the matrix. The direction in
which you examine the shapes depends on where you can most easily visualize the
relationship between the shapes.
The outer shape in each row remains the same, however it rotates 90-degrees
clockwise. Therefore, the shape that will appear in the missing box will be a pentagon,
and it will be pointing upwards (in order to match the rotation pattern).
In addition, in each row and column, the shapes are filled with a different number of
black dots. The top two rows have shapes with one, two and three dots. As the third
row includes only shapes with two and three dots, the missing box must have a shape
with one dot.
The only answer that follows those conditions is answer (A).
Answers (B) and (C) contain the correct shape with the correct number of dots on the
inside (one). However, the pentagons they contain are not a 90-degree clockwise
rotation of the box to the left, and therefore they are incorrect.
Answers (D) and (E) have three and two dots inside, respectively. Since the column
and row already contain shapes with three and two dots, these answer choices are
incorrect and can also be ruled out.

78. Explanation
The correct answer is (B).
Identify the image that completes the pattern by examining how the series of shapes
change across the rows and down the columns within the matrix. The direction in
which you examine the shapes depends on where you can most easily visualize the
relationship between the shapes.

90
Look at the first row and try to find a connection between the triangles and the square.
Notice that the square on the right can be created by putting the two triangles together
and removing the diagonal line on the side they share:

The same connection can be applied to the second row as the two triangles can form
the parallelogram on the right. Therefore, the missing box will contain a shape
comprised of the two shapes on the other two boxes. In addition, the shapes in each
column have the same amount of edges which means the empty box must contain a
shape with four edges (like the square and the parallelogram).
The correct answer is (B) as the triangles in this row combined can form:

79. Explanation
The correct answer is (B).
Identify the image that completes the pattern by examining how the series of shapes
change across the rows and down the columns within the matrix. The direction in
which you examine the shapes depends on where you can most easily visualize the
relationship between the shapes.
Each box contains a different geometrical shape. Along the columns, the shapes
number of sides increases by one. For example, in the left column the shape changes
from a triangle to a square and then a pentagon; 3, 4, and 5 sides, respectively. This
rule is enough to answer the question because the answer must have one more side
than the octagon, making the correct answer (B).
Side note: You can also notice that the number of sides in the rows increases by 2.

80. Explanation
The correct answer is (D).
Identify the image that completes the pattern by examining how the series of shapes
change across the rows and down the columns within the matrix. You can try and find
the pattern formed along the rows or down the columns as they both results the same
answer:
Along the rows - the boxes on the right column contain white areas in places that the
two other boxes on the same row contain white areas.
Down the columns - the boxes on the bottom row contain black areas in places that
the two other boxes on the same column contain black areas.
Using either of this methods will result the answer (D).

91
81. Explanation
The correct answer is (D).
Identify the image that completes the pattern by examining how the series of shapes
change across the rows and down the columns within the matrix. The direction in
which you examine the shapes depends on where you can most easily visualize the
relationship between the shapes.
Each row has the same shape with a different number of white dots. When moving
from left to right, the number of dots is divided by 2 (notice the number of dots is also
divided in each column when moving from top to bottom).
Therefore, the empty box must contain an arrow with exactly one dot (as two divided
by two is one), making answer (D) correct.
Side note: the location of the dots is random and only meant to distract from the
pattern.

82. Explanation
The correct answer is (C).
Identify the image that completes the pattern by examining how the series of shapes
change across the rows and down the columns within the matrix. The direction in
which you examine the shapes depends on where you can most easily visualize the
relationship between the shapes.
The first thing to notice is that each row contains a different pair of shapes. Therefore,
you can rule out answer (A) immediately, as it contains ellipses and not stars.
In each row, when moving from left to right the shapes move closer to each other,
while in the rightmost box the shared area is colored black.
The only answer choice that follows this pattern is answer (C), and therefore it is the
correct answer.
In answer (B) everything is colored except the shared area, thus it is incorrect.
In answers (D) and (E) nothing is colored; thus, they are also incorrect.

83. Explanation
The correct answer is (E).
Identify the image that completes the pattern by examining how the series of shapes
change across the rows and down the columns within the matrix. The direction in
which you examine the shapes depends on where you can most easily visualize the
relationship between the shapes.
The left most box in each row is comprised of the other boxes in the same row.
Similarly, the top box in each column is comprised of the other two boxes in that
column.
Therefore, the empty box must contain a shape which, together with the adjacent
boxes, will complete both the shapes in the top box of its column and the leftmost box
of its row. Answer (E) is the correct answer.

84. Explanation
The correct answer is (C).

92
Identify the image that completes the pattern by examining how the series of shapes
change across the rows and down the columns within the matrix. The direction in
which you examine the shapes depends on where you can most easily visualize the
relationship between the shapes.
Across each row the shape in the middle column is comprised from the objects which
appear in only one of the shapes adjacent to it (in the same row). Lines that appear in
both adjacent shapes are not included.
Similarly, in each column the shape in the middle row is comprised of the objects that
appear in only one of the shapes adjacent to it (in the same column). Lines that appear
in both adjacent shapes are not included.
Therefore, the correct answer is (C), as it is the only answer that completes the
pattern.

85. Explanation
The correct answer is 5.
In the top row, from left to right, the figure rotates and the arrow and circle swap
colors.
In the bottom row we should have the same relationship. In the left frame the arrow is
behind the circle. We can eliminate the 1st and 4th answer choices because they have
the arrow in front of the circle. We can eliminate the 2nd choice because the arrow
did not rotate, and the 3rd choice because the shapes did not swap colors.
We are left with the 5th choice as the correct answer.

86. Explanation
The correct answer is 3.

In the top row, from left to right, the number of black shapes swaps with
the number of circles (the two shapes and three circles in the left frame become three
shapes and two circles in the right frame).
In the bottom row we should have the same relationship. In the left frame we have
two circles, therefore in the right frame we should have only two shapes.
We can eliminate the 2nd and 5th answer choices.
In the left frame we have three shapes, therefore in the right frame we should have
three circles.
We can further eliminate the 1st and 4th choices.
We are left with the 3rd choice as the correct answer.

87. Explanation
The correct answer is 4.
In the top row, every arrow in the left frame goes with a particular figure in the right
frame.
 Arrow pointing right = white heart
 Arrow pointing down = grey smiley face
 Arrow pointing up = grey sun

93
In the bottom row we should have the same relationship. We can eliminate the 3rd
answer choice because it contains a grey heart and not a white heart.
We can also eliminate the 1st and 5th choices because they contain frowny faces and
not smiley faces.
Finally we can eliminate the 2nd answer choice because we should have a smiley face
in the upper-right corner (where there was a down arrow in the left frame), and a sun
in the upper-left and lower-right corners (where in the left frame we had up arrows),
but we have the opposite.
We are left with the 4th choice as the only correct answer.

88. Explanation
The correct answer is 1.
There are two changes that occur between the first two figures in the first part of the
analogy: the figure rotates clockwise 90°, and the color changes from black to
white.
The same changes must apply to the third figure to complete the analogy.
This means that answer 1 is the correct answer.

89. Explanation
The correct answer is 3.
The relationship between the first two figures is as follows: from the first frame to the
second frame, all the circles become rectangles, and one of the small rectangles
disappears.
The correct answer must have the same relationship with the third figure: it should be
a large rectangle with three small rectangles.
This means that 3 is the correct answer.

90. Explanation
The correct answer is 2.
The relationship between the first two figures is as follows:
 The bottom shape (trapezoid) in the first figure becomes the outermost
shape in the second figure;
 The middle shape (triangle) is rotated 90° clockwise and becomes
the innermost shape the second figure;
 The top shape (circle) becomes the middle shape in the second figure.
The correct answer should have the same relationship with the third figure.
The bottom shape (rectangle) in the third figure must become the outermost
shape. This means we can eliminate answers 4 and 5.
The middle shape (triangle) must be rotated 90° and become the innermost
shape, and the top shape (circle) must become the middle shape. Answers 1
and 3 can be eliminated, since they do not have the triangle in the correct
position, leaving 2 as the correct answer.

94
91. Explanation
The correct answer is 4.
The relationship between the first two figures is as follows:
- The inner shape (triangle) in the first figure moves to fill in the missing piece of
the outer shape.
- The inner shape (triangle) is rotated 90° counterclockwise.
- The inner shape (triangle) changes from white to black.
The correct answer will have the same relationship with the second figure. Since the
inner shape must move to the outside of the outer shape, we can eliminate answers 1
and 5. Answers 2 and 3 can also be eliminated since the outer shape is not supposed to
rotate.
This leaves 4 as the correct answer.

92. Explanation
The correct answer is 5.
The second figure is the same as first figure with two changes: the figure
becomes shaded, and the figure is extended in the direction of the dark area. The
correct answer should have the same relationship with the third figure.
This means that 5 is the correct answer.

93. Explanation
The correct answer is 2.
The relationship between the first two figures is as follows: the shapes in the first
frame each become shapes with one less side, the black shape becomes white and
the white shape becomes black. The correct answer must have the same relationship
with the third figure. The correct answer will be a large black triangle with a small
white four-sided shape.
This means that 2 is the correct answer.

94. Explanation
The correct answer is 4.
The relationship between the first two figures is as follows: the shapes in the first
frame each become shapes with one more side, the black shape becomes white and
the white shape becomes black. The correct answer must have the same relationship
with the third figure. The correct answer will be a large white pentagon with a small
black four-sided shape.
This means that 4 is the correct answer.

95. Explanation
The correct answer is 1.
The relationship between the first and second figure is as follows: the shape on the
left disappears, and the remaining shape on the right turns black. The correct
answer must have the same relationship with the third figure. This means that the
triangle on the left will disappear, and the remaining rectangle on the right will remain

95
and turn black.
This means that 1 is the correct answer.

96. Explanation
The correct answer is 3.
In the top row, from left to right, the lower half of the figure is removed. The same
should happen in the bottom row.
Since in the left frame we have two grey circles on top and two black circles on the
bottom, in the right frame we should only have the grey circles on top. We can
eliminate all the answer choices with black circles on the bottom (the 1st, 2nd, 4th,
and 5th choices).
We are left with the 3rd choice, which is the only correct answer

97. Explanation
The correct answer is 4.
In the top row, from left to right, the figure rotates 180° (one half of a circle – as if
standing on its head).
In the bottom row we should have the same relationship. We can eliminate the 2nd
and 3rd answer choices because the figure only rotated 90° (a quarter circle – as if
turned on its side). We can also eliminate the 1st and 5th choices because the
rectangles in the center of the figure are both the same color while they should be
different colors (one grey and one white).
We are left with the 4th choice as the only correct answer.

98. Explanation
The correct answer is 3.
Look closely at the top row from left to right:
In the left frame there is a circle with a vertical and a horizontal line inside. In the
next frame (on the right), the circle shrinks and, in the center, overlaps the lines.
The same pattern should occur in the bottom row:
In the left frame, there is a diamond with two diagonal lines. In the next frame, the
diamond should shrink and overlap the diagonal lines in the center of the frame.
Answer choice 1 is incorrect as the diamond is transparent and the lines can be seen.
Answer choice 2 is incorrect since the lines are no longer diagonal and the shape in
the middle is a circle instead of a diamond.
Answer choice 4 is incorrect since the lines are not diagonal and there is no shape in
the center.
Answer choices 5 is incorrect as it contains a black (not grey) diamond.
We are left with the 3rd choice, which is the only correct answer.

99. Explanation
The correct answer is 1.
Take a look at the top row. The order of the shapes in the left frame determines the
position of the shapes in the right frames.

96
 The top shape (the star) is positioned in the center,
 The middle shape (the circle) is positioned on the outside, and -
 The bottom shape (the triangle) is positioned in between the two objects.
The same rule will apply in the bottom row:
The circle sector will be in the center, so we can eliminate answer choices 4 and 5.
The plaque will be the outer shape, so we can eliminate answer choice 2.
The ring will be positioned in between both shapes. We can eliminate answer choice 3,
and are left with answer choice 1 as the correct answer.

100. Explanation
The correct answer is 2.

There are two rules connecting the two shapes. In the top row, from left to right:
1.The shape on the right has been rotated 180 degrees.
2.The shading has changed on the checked part of the shape. In the shape on the right,
the checked pattern, which was on the left shape, has been replaced by light blue
shading.

You need to look for a shape that follows these two rules for the pyramid shape.
Answer choices 1 and 3 have not been rotated correctly through 180 degrees, and
therefore they can be eliminated.

Using rule number 2, look at shapes 2, 4, and 5. In the original shape on the second
row, the patterned part was in the middle section, so look for a shape with light blue
shading in the middle section. Only shape 2 matches this. Check that the other shaded
parts have stayed the same. The white is still white and the dark blue is still dark blue.
Therefore, 2 is the correct answer.

101. Explanation
The correct answer is 4.

In the top row we have two figures - how do they go together?


We can see that the two figures look similar, but:
• The figure on the left is rotated on the right,
And two new parts have been added to the figure:
• A rounded dark shape behind it, and -
• A vertical line that runs in the middle of it.

In the bottom row we have a single figure, and we need to find a figure that will go
together with it in the same way that the two figures above go together.

We can immediately eliminate answer choice 2 because it belongs to the family of


figures in the top row, and not the bottom row.

97
We can also eliminate answer choice 3 because it is positioned in the same way as the
first figure, and is not rotated.
When we look at the other three answer choices, the only one that matches the pattern
is answer choice 4.
Notice that in answer choice 5 the vertical line is missing.
In answer choice 1 the dark shape in the back does not reach the middle of the main
shape, as in the top row.

Therefore the 4th answer choice is the correct answer.

102. Explanation
The correct answer is 3.

In the top row we have two figures - how do they go together?


We can see that the two figures look similar, but:
• On the right, the outer shape has received another side,
• The middle shape has been flipped vertically, and –
• The inner shape has become more rectangular.

In the bottom row we have a single figure, and we need to find a figure that will go
together with it in the same way that the two figures above go together.

We can immediately eliminate answer choice 5 because it belongs to the family of


figures in the top row, and not the bottom row.
We can also eliminate answer choice 2, because it has the same number of sides as the
first figure.
When we look at the other three answer choices, the only one that matches the pattern
is answer choice 3.
In answer choices 1 and 4 the figures are closed and not open.

Therefore the 3rd answer choice is the correct answer.

103. Explanation
The correct answer is 5.

In the top row, from left to right, the shapes move to the center of the frame
and swap colors, and the bottom shape flips.
In the bottom row we should have the same relationship.

We can eliminate the 1st and 2nd answer choices because the shapes did not swap
colors. We can eliminate the 4th choice as the upper shape (the bar) did not move to
the center of the frame. Finally we can eliminate the 3rd choice because the lower
shape (the star) did not flip.

98
We are left with the 5th choice as the only correct answer.

104. Explanation
The correct answer is 2.

In the top row, from left to right, the three arrows flip and the hearts swap colors.

In the bottom row we should have the same relationship.


We can eliminate the 4th answer choice because the hearts should be on the side.
We can eliminate the 1st and 3rd choices because the arrows should be pointing
towards the hearts, not away from them.
Finally we can eliminate the 5th answer choice because the hearts are the wrong
colors.

We are left with the 2nd choice as the correct answer.

105. Explanation
The correct answer is 5.

Look carefully at the change in the top row from left to right.
The background changes from grey to white, and each circle changes to a
square with the shaded part switching from left to right, or right to left (but not up
and down).

In the bottom row we should have a similar relationship. We can easily eliminate the
2nd answer choice because it contains circles (not squares), and the 4th choice
because the background is grey (not white). We can also eliminate the 1st choice
because it changed in the opposite way than how it was supposed to (the black section
rotated from its position in the left frame and switched sides from its position in the
frame above, while it should have done the opposite). Finally, we can eliminate the
3rd answer choice because the black sections moved in different directions for each
shape, and didn't follow any specific rule.

We are left with the 5th choice as the only correct answer.

106. Explanation
The correct answer is 2.

In the top row, from left to right, we added a duplicate shape (another arrow that
was flipped 180 degrees) over the original shape and the overlapping area became
black.

In the bottom row we should have the same relationship. Since in the left frame we

99
have a square ring shape, in the right frame the shape should be doubled, so we can
eliminate the 1st answer choice. Further, only the overlapping areas should be black,
so we can also eliminate the 3rd, 4th and 5th choices.

We are left with the 2nd answer choice as the only correct answer.

107. Explanation
The correct answer is 1.

The relationship between the first two figures is as follows: the second figure
represents all of the lines which are shared by the two shapes in the first figure.
Each of the two shapes in the first figure has a diagonal line which is not shared with
the other. These lines disappear in the second figure.
The correct answer must have the same relationship with the third figure.

This means that 1 is the correct answer.

108. Explanation
The correct answer is 3.
In the top row, the left frame contains a circle, two diagonal lines, and a vertical line.
The right frame contains only the diagonal lines.
The same relationship will hold for the bottom row.
The answer choice will contain only the three diagonal lines present in the bottom-left
frame.
Therefore, answer choice 3 is the correct answer.

109. Explanation
The correct answer is 2.
When we move from left to right across:
 In the top row, the color of the background of the frames changes from black
to white. The same change should occur in the bottom row.
 In the top row, the shapes at the bottom (the circle and the square) switch
sides. The same change should occur in the bottom row, and the triangle and
the rhombus should switch sides.
 In the top row, the shape on the bottom right (the circle) turns from grey to
black. In the bottom row the triangle should turn from gray to black.
 In the top row, the shape at the top (the triangle) rotates 180 degrees. The
same change should occur in the bottom row, and the ¾ circle should rotate
180 degrees.
The only answer choice which follows all rules is the 2nd

110. Explanation
The correct answer is 5.
In the top row we have two figures - how do they go together?

100
The shape on the right is the same as the figure on the left, rotated 180 degrees.

In the bottom row we have a single figure, and we need to find a figure that will go
together with it in the same way that the two figures above go together.
The only answer which fills the rule is the 5th.
Therefore, the correct answer is 5.

111.0Explanation
The correct answer is 3.
In the top row we have two figures - how do they go together?
As we move from left to right:

• The white triangles change into black diamonds,


• The hearts rotate 180 degrees but remain gray,
• The number of circles increases by one, and -
• The circles change colors from black to white
In the bottom row, we have a single figure, and we need to find a figure that will go
together with it in the same way that the two figures above go together.
We can eliminate answer choice 2 as it contains a white diamond and not a black one.
In addition, this answer choice contains black circles instead of white ones.
We can also eliminate answer choice 1 as it contains a white heart and not a gray one.
We can eliminate answer choice 4 as it contains three circles instead of four.
Finally, we can eliminate answer choice 5 as it contains a heart that has not been
rotated.
We are left with answer choice 3, which is the correct answer.

112. Explanation
The correct answer is 1.
In the top row we have two figures - how do they go together?
As we move from left to right:
• The inner and outer shapes have switched places – on the right the diamond is now
the outer shape and the circle is now the inner shape.
• The inner shapes contain shaded segments in the same places, but the shading has
changed from gray to brick-textured.
• The outer shapes contain shaded segments in opposite places and the shading has
changed from brick-textured to gray.
In the bottom row we have a single figure, and we need to find a figure that will go
together with it in the same way that the two figures above go together.
Answer choices 2, 3 and 5 don’t have shading in all segments of the inner and outer
shapes, so they can be eliminated.
In answer choice 4 the gray segments of the inner shape have remained gray in the
figure on the right, so we can eliminate it as well.
We are left with answer choice 1, which is the correct answer.

101
113. Explanation
The correct answer is 4.
In this question, from left to right, we can see that:

 In the top row, the outer shape (trapezoid) becomes a shape with one less
side (triangle). Thus, in the bottom row, the outer shape (rectangle) should also
change into a shape with one less side (triangle).
 In the top row, the inner shape (pentagon) becomes a shape with one less
side (rectangle). Thus, in the bottom row, the inner shape (hexagon) should
also change into a shape with one less side (a pentagon).
 The color of the outer shape fades out from top to bottom instead of bottom
to top.
 The inner shape changes its color from black to white.
The 4th answer choice follows all the above conditions. Therefore, this is the correct
answer.
The 2nd answer choice is incorrect as the color of the outer shape should fade out
from top to bottom, not from bottom to top.

114. Explanation
The correct answer is 2.
In this question, from left to right, we can see that:

 In the top row, the whole shape (parallelogram) becomes black. Thus, in the
bottom row, the whole shape (heart) should also become black.
 In the top row, a white arrow is added to the right-hand figure. It points in the
direction of where the dark area in the left-hand figure was located (the
right-bottom corner of the parallelogram). Thus, in the bottom row, a white
arrow pointing to where the left-hand figure's dark area was located (the
bottom corner of the heart) should be added to the right-hand figure.
The 2nd answer choice follows all the above conditions. Therefore, this is the correct
answer.

115. Explanation
The correct answer is 2.
In the top row, there are two figures — how do they go together?
It seems that in the figure on the right, only the bottom triangle stays.
Thus, in the bottom row, let’s look for an answer choice that is the same as the bottom
diamond.
We can see that there is no such answer choice. Therefore, we need to think of
another way the two figures in the top row go together.
If we look closely at the right figure in the top row, we can see that it consists of only
the lines shared by both triangles on the left. Both triangles on the left have two
vertical lines. However, the top triangle has one additional horizontal line which the
bottom triangle doesn't have. Thus, in the figure on the right, this horizontal line

102
disappears.

In the bottom row, we should have the same relationship. The correct answer should
consist of only the lines shared by both diamonds on the left. Both diamonds on the
left have one horizontal line. However, the bottom diamond has one additional short
diagonal line which the top diamond doesn't have. Thus, in the figure on the right, this
short diagonal line should disappear.

The 2nd answer choice follows this condition. Therefore, the correct answer is 2.

116. Explanation
The correct answer is (B).
Identify the image that completes the pattern by examining how the series of shapes
change across the rows and down the columns within the matrix. The direction in
which you examine the shapes depends on where you can most easily visualize the
relationship between the shapes.
When you look at each row from left to right, notice the shape lose a line. When
moving from the leftmost box to the middle one, the shape loses its vertical line, and
when moving from the middle box to the rightmost box the shape loses the horizontal
line, leaving only the diagonal lines.
When moving from top to bottom in each column the shapes lose a line. When
moving from the top box to the middle box the shape loses the diagonals which
stretch from top-right to bottom-left, and when moving from the middle box to the
bottom the shape loses the other diagonal (which stretches from top-left to
bottom-right).
Answer (B) is the only answer which follows the pattern (the empty box), and
therefore it is the correct answer.
Alternative Solution: When moving either left to right or top to bottom, the shape
loses exactly one line. Therefore, the answer box must be empty to meet this
condition.

117. Explanation
The correct answer is (A).
Identify the image that completes the pattern by examining how the series of shapes
change across the rows and down the columns within the matrix. The direction in
which you examine the shapes depends on where you can most easily visualize the
relationship between the shapes.
Each column has a different kind of background. In addition, across each row (when
moving from left to right) the position of the black dot changes and moves clockwise
along the corners of the shape.
The correct answer is (A), since it is the only answer choice which follows the
pattern.
Answer choices (B), (C) and (E) can be ruled out as their background is different than
the background shown in the column of the missing box.

103
The dot in answer (D) is misplaced, making this answer choice incorrect.

118. Explanation
The correct answer is (B).
Identify the image that completes the pattern by examining how the series of shapes
change across the rows and down the columns within the matrix. The direction in
which you examine the shapes depends on where you can most easily visualize the
relationship between the shapes.
Across each row the box on the right contains a quotient arrived at by dividing the
other two numbers in the same row.
Along each column the box at the bottom is the product of multiplying the other two
boxes in that column.
Therefore, the answer is (B), as along its row 6 ÷ 2 = 3 and down that column 1 × 3 =
3.

119. Explanation
The correct answer is (C).
Identify the image that completes the pattern by examining how the series of shapes
change across the rows and down the columns within the matrix. The direction in
which you examine the shapes depends on where you can most easily visualize the
relationship between the shapes.
In each row the box on the right sums the number of white circles in the other boxes
in its row. Similarly, the box on the bottom in each column sums the number of black
circles in its column.
Therefore, the empty box should sum the number of black circles in its column (four)
and white circles in its row (four), making answer (E) correct.

120. Explanation
The correct answer is (A).
Identify the image that completes the pattern by examining how the series of shapes
change across the rows and down the columns within the matrix. The direction in
which you examine the shapes depends on where you can most easily visualize the
relationship between the shapes.
Across each row the image is rotated 90-degrees clockwise. Therefore, the empty box
will contain a shape similar to the box on its left, but rotated, making answer (A)
correct.

121. Explanation
The correct answer is (C).
Identify the image that completes the pattern by examining how the series of shapes
change across the rows and down the columns within the matrix. The direction in
which you examine the shapes depends on where you can most easily visualize the
relationship between the shapes.

104
When going through the pictures in each row the shapes rotate counter-clockwise, and
when moving down each column (from top to bottom) the shapes rotate clockwise,
while in the bottom row the stars are deleted.
Answer (C) is the correct answer as it fits the pattern. It does not include stars, the
shapes inside are a clockwise rotated version of the box above it, and a
counter-clockwise rotated version of the box to its left.

122. Explanation
The correct answer is (E).
Identify the image that completes the pattern by examining how the series of shapes
change across the rows and down the columns within the matrix. The direction in
which you examine the shapes depends on where you can most easily visualize the
relationship between the shapes.
Across each row the white star moves from left to right. Down each column the white
star moves from up to down; in the top row it is positioned above the black stars, in
the middle row it is positioned in between them, and in the bottom row it is positioned
below them. The empty box is on the bottom row; therefore, the white star will be
below the black stars and it is also in on the right column, thus the star will appear on
the right. Therefore, the correct answer is (E).

123. Explanation
The correct answer is (D).
Identify the image that completes the pattern by examining how the series of shapes
change across the rows and down the columns within the matrix. The direction in
which you examine the shapes depends on where you can most easily visualize the
relationship between the shapes.
Every box contains the same shapes – a square, a triangle and a circle, and one of
those shapes is patterned with diagonal lines. You can notice that each column has the
exact same shape, thus the empty box will contain a shape similar to its column.
Therefore, you can rule out answers (B), (C) and (E).
In addition, when moving across each row the patterned shape is either the inner,
middle or outer one: The top row has the outer shape colored, the colored shape in the
middle row is the one in the middle, and the bottom row has the innermost colored.
As the empty box is in the last row, the patterned shape will be the inner shape.
Therefore, the correct answer is (D).

124. Explanation
The correct answer is 3.

In the top row we have two figures - how do they go together?


We can see that the two figures look similar, but:

• The figure on the left is flipped horizontally on the right.


• Also, the circle is now dark and has moved behind the shape.

105
Two new shapes have been added to the figure:
• A vertical line next to the dark circle, and -
• A white circle in the corner above it.

In the bottom row, we have a single figure, and we need to find a figure that will go
together with it in the same way that the two figures above go together.

We can immediately eliminate answer choice 4 because it belongs to the family of


figures in the top row, and not the bottom row.
We can also eliminate answer choice 2, because it is positioned in the same way as
the first shape, and is not flipped.
When we look at the other three answer choices, we see that the only one that matches
the pattern is answer choice 3.
Notice that answer choice 5 includes nearly all the changes we need, but it is a closed
shape and not an open one like the figures in the question.

Therefore the 3rd answer choice is the correct answer.

125. Explanation
The correct answer is 1.

In the top row we have two figures - how do they go together?


We can see that the two figures look similar, but they some changes have occurred
from the left side to the right:
• The outer shape has received one extra side, making it into a new shape
• The middle shape's rounded side has become straight sides with rounded corners
• The inner shape has been flipped on its side
• All the shapes have been closed (the missing side has been added without further
changing the shape)

In the bottom row, we have a single figure, and we need to find a figure that will go
together with it in the same way that the two figures above go together.

The only three choices that change the outer shape to one with one more side are 1, 4,
and 5.
Answer choice 1 makes all four changes listed above.
Answer choice 4 does not close the shapes.
Answer choice 5 does not flip the inner shape on its side.

Therefore, the 1st answer choice is the correct answer.

126. Explanation
The correct answer is 4.

106
In the top row we have two figures - how do they go together?

We can see that the two figures look similar, but:


• On the right, the figure is flipped horizontally,
• The long vertical line has been removed,
• The short vertical line now runs through entire length of the shape, and -
• A new horizontal line has been added.

In the bottom row we have a single figure, and we need to find a figure that will go
together with it in the same way that the two figures above go together.

We can immediately eliminate answer choice 3 because it looks like the shape in the
top row, and not the bottom row.
We can also eliminate answer choices 1 and 5, because they have two vertical lines,
while we only need one.
When we look at the other two answer choices, the only one that matches the pattern
is answer choice 4.
In answer choice 2 there are two horizontal lines while we need only one.

Therefore the 4th answer choice is the correct answer.

127. Explanation
The correct answer is 2.

The relationship between the first two figures is as follows: the second figure is the
same as the first figure, but rotated 90° counterclockwise, and with a triangle
added on top. The correct answer must have the same relationship with the third
figure.

This means that 2 is the correct answer.

128. Explanation
The correct answer is 2.

Look closely at the top row: As you move across the row, the star is doubled and
is partially overlapping. The parts which overlap become black.

Apply the same rule to the bottom row. As you double and partially overlap the
frames, only segments of the top and bottom of the frames will overlap and therefore
become black.

The only answer choice which follows this rule is 2.

107
129. Explanation
The correct answer is 4.

Take a look at the above image:


Notice that the shapes that have the same frame correspond with each other and move
in a similar way across the rows:

•The red rectangle on the left of the box moves to the right, adjacent to the wall of
the box,
•The blue rectangle on the right of the box moves to the left, and
•The green rectangle moves from behind the blue rectangle to behind the red
rectangle.

The same pattern should occur in the bottom row. The only answer choice that
follows these rules is the 4th.

130. Explanation
The correct answer is 1.

In the top row, when we move from the left box to the right box, the diagonal
lines are removed and the vertical and horizontal lines remain.

When we look at the answer choices to find the one which will have the same
relationship with the left box in the bottom row, the only answer which follows these
rules is the 1st.

108
131. Explanation
The correct answer is 3.

In the top row we have two figures - how do they go together?


As we move from left to right:
•The shapes change from triangles to circles,
•The number of shapes and their position remain the same,yet -
•All the shapes but one change color from white to black.

In the bottom row we have a single figure, and we need to find a figure that will go
together with it in the same way that the two figures above go together.

In answer choice 5 we have three circles and not two, so it can be eliminated.
In answer choices 1 and 4 none of the circles have changed color, so we can eliminate
them as well.
In answer choice 2 the shapes have changed position, so it is not a correct answer
either.
We are left with answer choice 3, which is the correct answer.

132. Explanation
The correct answer is 5.
In the top row we have two figures - how do they go together?
As we move from left to right:
The shapes on the right mirror the shapes on the left,
The circle moves in front of the triangle,
•The background color changes from gray to black,
•The triangle changes from black to gray, and –
•The circle changes from gray to white.
In the bottom row we have a single figure, and we need to find a figure that will go
together with it in the same way that the two figures above go together.

Answer choice 2 can be eliminated because the background hasn’t changed to black.
Answer choice 3 can be eliminated because the circle remained at the back of the
triangle.
Answer choice 4 can be eliminated because the circle has move higher than in the
figure on the left.
Answer choice 1 can be eliminated because the square has moved to the back of the
triangle while it should have remained in place.

Answer choice 5 is the only one that follows all of the above rules and is the correct
answer.

109
133. Explanation
The correct answer is 1.
In the top row, from left to right, the figure changes as follows:
 A mirror image of the pentagon is added, forming an upside down pentagon
attached to the original one from below.
 The original pentagon changes its color from white to black.

In the bottom row, we should have the same relationship: A mirror image of the
triangle should be added to form an upside down triangle. This upside down triangle
should be attached to the original triangle from below. In addition, the original
triangle should change its color from white to black. Therefore, the 1st choice is the
correct answer.
The 5th choice is incorrect as the correct relationship between the figures does not
involve flipping the original figure from right to left.

134. Explanation
The correct answer is 2.

In the top row, from left to right, the figure changes as follows:

•The top shape (circle) becomes the middle shape in the second figure.
•The middle shape (pentagon) becomes the outermost shape in the second figure.
•The bottom shape (arrow) rotates 90° counterclockwise, changes its color to blue,
and becomes the innermost shape in the second figure.

In the bottom row, there should be the same relationship:

•The top shape (circle) should become the middle shape.


•The middle shape (pentagon) should become the outermost shape.
•The bottom shape (moon) should be rotated 90° counterclockwise, colored in
with blue, and become the innermost shape.

The second answer choice follows all the above conditions. Therefore, this is the
correct answer.

The third answer choice is incorrect as the moon inside this figure is rotated 90°
clockwise, instead of 90° counterclockwise.

Alternatively, the bottom shape rotates 90 degrees counterclockwise, changes to blue,


and then moves inside the top shape.
Next, the top shapes move into the middle shape.

110
135. Explanation
The correct answer is 5.

In this question, from left to right, we can see that:

 In the top row, the left-top shape (the smiley face) is flipped over
upside-down but stays in its place. Thus, in the bottom row, the left-top
shape (the heart) should also be flipped over upside-down but stay in its place.
 In the top row, the left-bottom shape (the arrow) stays in its place and does
not change at all. Thus, in the bottom row, the left-bottom shape (the
trapezoid) should also stay in its place.
 In the top row, the right-top shape (the “L-shape”) and the right-bottom shape
(the triangle) switch places. Additionally, the new right-top shape (the
triangle) changes its color from white to black. Thus, in the bottom row, the
right-top shape (the star) and the right-bottom shape (the arc) should switch
places, and the new right-top shape (the arc) should change its color from
white to black.

The 5th answer choice follows all the above conditions. Therefore, this is the correct
answer.

136. Explanation
The correct answer is 3.
In this question, from left to right, we can see that:
 In the top row, shapes are inserted into the outer shape (triangle) to create the
figure on the right. The number of these inner shapes is the same as
the number of the outer shape’s sides: In the top row, the outer shape
has three sides, and thus three shapes are inserted. In the bottom row, the
outer shape has four sides, and thus four shapes should be inserted.
 In the top row, each of the newly inserted shapes has one side more than the
outer shape. The outer shape (triangle) has three sides, and each of the inserted
shapes has four sides (squares). In the bottom row, the outer shape (square)
has four sides, and thus each of the inserted shapes should have five sides
(pentagons).
 The inserted shapes are colored with blue.
The 3rd answer choice follows all the above conditions. Therefore, this is the
correct answer.

137. Explanation
The correct answer is (A).
Identify the image that completes the pattern by examining how the series of shapes
change across the rows and down the columns within the matrix. The direction in
which you examine the shapes depends on where you can most easily visualize the
relationship between the shapes.

111
The outer shape in each row remains the same, however it rotates 90-degrees
clockwise. Therefore, the shape that will appear in the missing box will be a pentagon,
and it will be pointing upwards (in order to match the rotation pattern).
In addition, in each row and column, the shapes are filled with a different number of
black dots. The top two rows have shapes with one, two and three dots. As the third
row includes only shapes with two and three dots, the missing box must have a shape
with one dot.
The only answer that follows those conditions is answer (A).
Answers (B) and (C) contain the correct shape with the correct number of dots on the
inside (one). However, the pentagons they contain are not a 90-degree clockwise
rotation of the box to the left, and therefore they are incorrect.
Answers (D) and (E) have three and two dots inside, respectively. Since the column
and row already contain shapes with three and two dots, these answer choices are
incorrect and can also be ruled out.

138. Explanation
The correct answer is (B).
Identify the image that completes the pattern by examining how the series of shapes
change across the rows and down the columns within the matrix. The direction in
which you examine the shapes depends on where you can most easily visualize the
relationship between the shapes.
Look at the first row and try to find a connection between the triangles and the square.
Notice that the square on the right can be created by putting the two triangles together
and removing the diagonal line on the side they share:

The same connection can be applied to the second row as the two triangles can form
the parallelogram on the right. Therefore, the missing box will contain a shape
comprised of the two shapes on the other two boxes. In addition, the shapes in each
column have the same amount of edges which means the empty box must contain a
shape with four edges (like the square and the parallelogram).
The correct answer is (B) as the triangles in this row combined can form:

139. Explanation
The correct answer is (B).
Identify the image that completes the pattern by examining how the series of shapes
change across the rows and down the columns within the matrix. The direction in

112
which you examine the shapes depends on where you can most easily visualize the
relationship between the shapes.
Each box contains a different geometrical shape. Along the columns, the shapes
number of sides increases by one. For example, in the left column the shape changes
from a triangle to a square and then a pentagon; 3, 4, and 5 sides, respectively. This
rule is enough to answer the question because the answer must have one more side
than the octagon, making the correct answer (B).
Side note: You can also notice that the number of sides in the rows increases by 2.

140. Explanation
The correct answer is (D).
Identify the image that completes the pattern by examining how the series of shapes
change across the rows and down the columns within the matrix. You can try and find
the pattern formed along the rows or down the columns as they both results the same
answer:
Along the rows - the boxes on the right column contain white areas in places that the
two other boxes on the same row contain white areas.
Down the columns - the boxes on the bottom row contain black areas in places that
the two other boxes on the same column contain black areas.
Using either of this methods will result the answer (D).

141. Explanation
The correct answer is (D).
Identify the image that completes the pattern by examining how the series of shapes
change across the rows and down the columns within the matrix. The direction in
which you examine the shapes depends on where you can most easily visualize the
relationship between the shapes.
Each row has the same shape with a different number of white dots. When moving
from left to right, the number of dots is divided by 2 (notice the number of dots is also
divided in each column when moving from top to bottom).
Therefore, the empty box must contain an arrow with exactly one dot (as two divided
by two is one), making answer (D) correct.
Side note: the location of the dots is random and only meant to distract from the
pattern.

142. Explanation
The correct answer is (C).
Identify the image that completes the pattern by examining how the series of shapes
change across the rows and down the columns within the matrix. The direction in
which you examine the shapes depends on where you can most easily visualize the
relationship between the shapes.
The first thing to notice is that each row contains a different pair of shapes. Therefore,
you can rule out answer (A) immediately, as it contains ellipses and not stars.

113
In each row, when moving from left to right the shapes move closer to each other,
while in the rightmost box the shared area is colored black.
The only answer choice that follows this pattern is answer (C), and therefore it is the
correct answer.
In answer (B) everything is colored except the shared area, thus it is incorrect.
In answers (D) and (E) nothing is colored; thus, they are also incorrect.

143. Explanation
The correct answer is (E).
Identify the image that completes the pattern by examining how the series of shapes
change across the rows and down the columns within the matrix. The direction in
which you examine the shapes depends on where you can most easily visualize the
relationship between the shapes.
The left most box in each row is comprised of the other boxes in the same row.
Similarly, the top box in each column is comprised of the other two boxes in that
column.
Therefore, the empty box must contain a shape which, together with the adjacent
boxes, will complete both the shapes in the top box of its column and the leftmost box
of its row. Answer (E) is the correct answer.

144. Explanation
The correct answer is (C).
Identify the image that completes the pattern by examining how the series of shapes
change across the rows and down the columns within the matrix. The direction in
which you examine the shapes depends on where you can most easily visualize the
relationship between the shapes.
Across each row the shape in the middle column is comprised from the objects which
appear in only one of the shapes adjacent to it (in the same row). Lines that appear in
both adjacent shapes are not included.
Similarly, in each column the shape in the middle row is comprised of the objects that
appear in only one of the shapes adjacent to it (in the same column). Lines that appear
in both adjacent shapes are not included.
Therefore, the correct answer is (C), as it is the only answer that completes the
pattern.
Alternatively, you could look at the column on the right and the bottom row. The
shapes in the column on the right include objects that appear in only one of the shapes
in the same row. If an object appears in more than one other shape, it will not appear
in the shape on the right column.
Likewise, the shapes in the bottom row include objects that appear in only one of the
shapes in that column. If an object appears in more than one other shape, it will not
appear in the shape on the bottom row.

114
145. Explanation
The correct answer is 2.

There are two rules connecting the two shapes. In the top row, from left to right:
1.The shape on the right has been rotated 180 degrees.
2.The shading has changed on the checked part of the shape. In the shape on the right,
the checked pattern, which was on the left shape, has been replaced by light blue
shading.

You need to look for a shape that follows these two rules for the pyramid shape.
Answer choices 1 and 3 have not been rotated correctly through 180 degrees, and
therefore they can be eliminated.

Using rule number 2, look at shapes 2, 4, and 5. In the original shape on the second
row, the patterned part was in the middle section, so look for a shape with light blue
shading in the middle section. Only shape 2 matches this. Check that the other shaded
parts have stayed the same. The white is still white and the dark blue is still dark blue.
Therefore, 2 is the correct answer.

146. Explanation
The correct answer is 3.

In the top row we have two figures - how do they go together?


We can see that the two figures look similar, but:
• On the right, the outer shape has received another side,
• The middle shape has been flipped vertically, and –
• The inner shape has become more rectangular.

In the bottom row we have a single figure, and we need to find a figure that will go
together with it in the same way that the two figures above go together.

We can immediately eliminate answer choice 5 because it belongs to the family of


figures in the top row, and not the bottom row.
We can also eliminate answer choice 2, because it has the same number of sides as the
first figure.
When we look at the other three answer choices, the only one that matches the pattern
is answer choice 3.
In answer choices 1 and 4 the figures are closed and not open.

Therefore the 3rd answer choice is the correct answer.

147. Explanation
The correct answer is 5.

115
In the top row, from left to right, the shapes move to the center of the frame
and swap colors, and the bottom shape flips.
In the bottom row we should have the same relationship.

We can eliminate the 1st and 2nd answer choices because the shapes did not swap
colors. We can eliminate the 4th choice as the upper shape (the bar) did not move to
the center of the frame. Finally we can eliminate the 3rd choice because the lower
shape (the star) did not flip.

We are left with the 5th choice as the only correct answer.

148. Explanation
The correct answer is 2.

In the top row, from left to right, the three arrows flip and the hearts swap colors.

In the bottom row we should have the same relationship.


We can eliminate the 4th answer choice because the hearts should be on the side.
We can eliminate the 1st and 3rd choices because the arrows should be pointing
towards the hearts, not away from them.
Finally we can eliminate the 5th answer choice because the hearts are the wrong
colors.

We are left with the 2nd choice as the correct answer.

149. Explanation
The correct answer is 5.

Look carefully at the change in the top row from left to right.
The background changes from grey to white, and each circle changes to a
square with the shaded part switching from left to right, or right to left (but not up
and down).

In the bottom row we should have a similar relationship. We can easily eliminate the
2nd answer choice because it contains circles (not squares), and the 4th choice
because the background is grey (not white). We can also eliminate the 1st choice
because it changed in the opposite way than how it was supposed to (the black section
rotated from its position in the left frame and switched sides from its position in the
frame above, while it should have done the opposite). Finally, we can eliminate the
3rd answer choice because the black sections moved in different directions for each
shape, and didn't follow any specific rule.

We are left with the 5th choice as the only correct answer.

116
150. Explanation
The correct answer is 2.

In the top row, from left to right, we added a duplicate shape (another arrow that
was flipped 180 degrees) over the original shape and the overlapping area became
black.

In the bottom row we should have the same relationship. Since in the left frame we
have a square ring shape, in the right frame the shape should be doubled, so we can
eliminate the 1st answer choice. Further, only the overlapping areas should be black,
so we can also eliminate the 3rd, 4th and 5th choices.

We are left with the 2nd answer choice as the only correct answer.

151. Explanation
The correct answer is 1.

The relationship between the first two figures is as follows: the second figure
represents all of the lines which are shared by the two shapes in the first figure.
Each of the two shapes in the first figure has a diagonal line which is not shared with
the other. These lines disappear in the second figure.
The correct answer must have the same relationship with the third figure.

This means that 1 is the correct answer.

152. Explanation
The correct answer is 2.

When we move from left to right across:


 In the top row, the color of the background of the frames changes from black
to white. The same change should occur in the bottom row.
 In the top row, the shapes at the bottom (the circle and the square) switch
sides. The same change should occur in the bottom row, and the triangle and
the rhombus should switch sides.
 In the top row, the shape on the bottom right (the circle) turns from grey to
black. In the bottom row the triangle should turn from gray to black.
 In the top row, the shape at the top (the triangle) rotates 180 degrees. The
same change should occur in the bottom row, and the ¾ circle should rotate
180 degrees.
The only answer choice which follows all rules is the 2nd.

117
153. Explanation
The correct answer is 3.

In the top row we have two figures - how do they go together?


As we move from left to right:

• The white triangles change into black diamonds,


• The hearts rotate 180 degrees but remain gray,
• The number of circles increases by one, and -
• The circles change colors from black to white
In the bottom row, we have a single figure, and we need to find a figure that will go
together with it in the same way that the two figures above go together.

We can eliminate answer choice 2 as it contains a white diamond and not a black one.
In addition, this answer choice contains black circles instead of white ones.
We can also eliminate answer choice 1 as it contains a white heart and not a gray one.
We can eliminate answer choice 4 as it contains three circles instead of four.
Finally, we can eliminate answer choice 5 as it contains a heart that has not been
rotated.

We are left with answer choice 3, which is the correct answer.

154. Explanation
The correct answer is 1.

In the top row we have two figures - how do they go together?


As we move from left to right:
• The inner and outer shapes have switched places – on the right the diamond is now
the outer shape and the circle is now the inner shape.
• The inner shapes contain shaded segments in the same places, but the shading has
changed from gray to brick-textured.
• The outer shapes contain shaded segments in opposite places and the shading has
changed from brick-textured to gray.

In the bottom row we have a single figure, and we need to find a figure that will go
together with it in the same way that the two figures above go together.

Answer choices 2, 3 and 5 don’t have shading in all segments of the inner and outer
shapes, so they can be eliminated.
In answer choice 4 the gray segments of the inner shape have remained gray in the
figure on the right, so we can eliminate it as well.

We are left with answer choice 1, which is the correct answer.

118
155. Explanation
The correct answer is 4.

In this question, from left to right, we can see that:

 In the top row, the outer shape (trapezoid) becomes a shape with one less
side (triangle). Thus, in the bottom row, the outer shape (rectangle) should also
change into a shape with one less side (triangle).
 In the top row, the inner shape (pentagon) becomes a shape with one less
side (rectangle). Thus, in the bottom row, the inner shape (hexagon) should
also change into a shape with one less side (a pentagon).
 The color of the outer shape fades out from top to bottom instead of bottom
to top.
 The inner shape changes its color from black to white.

The 4th answer choice follows all the above conditions. Therefore, this is the correct
answer.

The 2nd answer choice is incorrect as the color of the outer shape should fade out
from top to bottom, not from bottom to top.

156. Explanation
The correct answer is 2.

In this question, from left to right, we can see that:

 In the top row, the whole shape (parallelogram) becomes black. Thus, in the
bottom row, the whole shape (heart) should also become black.
 In the top row, a white arrow is added to the right-hand figure. It points in the
direction of where the dark area in the left-hand figure was located (the
right-bottom corner of the parallelogram). Thus, in the bottom row, a white
arrow pointing to where the left-hand figure's dark area was located (the
bottom corner of the heart) should be added to the right-hand figure.
The 2nd answer choice follows all the above conditions. Therefore, this is the correct
answer.

157. Explanation
The correct answer is 2.

In the top row, there are two figures — how do they go together?
It seems that in the figure on the right, only the bottom triangle stays.
Thus, in the bottom row, let’s look for an answer choice that is the same as the bottom
diamond.

119
We can see that there is no such answer choice. Therefore, we need to think of
another way the two figures in the top row go together.

If we look closely at the right figure in the top row, we can see that it consists of only
the lines shared by both triangles on the left. Both triangles on the left have two
vertical lines. However, the top triangle has one additional horizontal line which the
bottom triangle doesn't have. Thus, in the figure on the right, this horizontal line
disappears.

In the bottom row, we should have the same relationship. The correct answer should
consist of only the lines shared by both diamonds on the left. Both diamonds on the
left have one horizontal line. However, the bottom diamond has one additional short
diagonal line which the top diamond doesn't have. Thus, in the figure on the right, this
short diagonal line should disappear.

The 2nd answer choice follows this condition. Therefore, the correct answer is 2.

158. Explanation
The correct answer is (A).
Identify the image that completes the pattern by examining how the series of shapes
change across the rows and down the columns within the matrix. The direction in
which you examine the shapes depends on where you can most easily visualize the
relationship between the shapes.
Each column has a different kind of background. In addition, across each row (when
moving from left to right) the position of the black dot changes and moves clockwise
along the corners of the shape.
The correct answer is (A), since it is the only answer choice which follows the
pattern.
Answer choices (B), (C) and (E) can be ruled out as their background is different than
the background shown in the column of the missing box.
The dot in answer (D) is misplaced, making this answer choice incorrect.

159. Explanation
The correct answer is (C).
Identify the image that completes the pattern by examining how the series of shapes
change across the rows and down the columns within the matrix. The direction in
which you examine the shapes depends on where you can most easily visualize the
relationship between the shapes.
In each row the box on the right sums the number of white circles in the other boxes
in its row. Similarly, the box on the bottom in each column sums the number of black
circles in its column.
Therefore, the empty box should sum the number of black circles in its column (four)
and white circles in its row (four), making answer (E) correct.

120
160. Explanation
The correct answer is (A).
Identify the image that completes the pattern by examining how the series of shapes
change across the rows and down the columns within the matrix. The direction in
which you examine the shapes depends on where you can most easily visualize the
relationship between the shapes.
Across each row the image is rotated 90-degrees clockwise. Therefore, the empty box
will contain a shape similar to the box on its left, but rotated, making answer (A)
correct.

161. Explanation
The correct answer is (C).
Identify the image that completes the pattern by examining how the series of shapes
change across the rows and down the columns within the matrix. The direction in
which you examine the shapes depends on where you can most easily visualize the
relationship between the shapes.
When going through the pictures in each row the shapes rotate counter-clockwise, and
when moving down each column (from top to bottom) the shapes rotate clockwise,
while in the bottom row the stars are deleted.
Answer (C) is the correct answer as it fits the pattern. It does not include stars, the
shapes inside are a clockwise rotated version of the box above it, and a
counter-clockwise rotated version of the box to its left.

162. Explanation
The correct answer is (E).
Identify the image that completes the pattern by examining how the series of shapes
change across the rows and down the columns within the matrix. The direction in
which you examine the shapes depends on where you can most easily visualize the
relationship between the shapes.
Across each row the white star moves from left to right. Down each column the white
star moves from up to down; in the top row it is positioned above the black stars, in
the middle row it is positioned in between them, and in the bottom row it is positioned
below them. The empty box is on the bottom row; therefore, the white star will be
below the black stars and it is also in on the right column, thus the star will appear on
the right. Therefore, the correct answer is (E).

163. Explanation
The correct answer is (D).
Identify the image that completes the pattern by examining how the series of shapes
change across the rows and down the columns within the matrix. The direction in
which you examine the shapes depends on where you can most easily visualize the
relationship between the shapes.
Every box contains the same shapes – a square, a triangle and a circle, and one of
those shapes is patterned with diagonal lines. You can notice that each column has the

121
exact same shape, thus the empty box will contain a shape similar to its column.
Therefore, you can rule out answers (B), (C) and (E).
In addition, when moving across each row the patterned shape is either the inner,
middle or outer one: The top row has the outer shape colored, the colored shape in the
middle row is the one in the middle, and the bottom row has the innermost colored.
As the empty box is in the last row, the patterned shape will be the inner shape.
Therefore, the correct answer is (D).

164. Explanation
The correct answer is 5.

In the top row, from left to right, the figure rotates and the arrow and circle swap
colors.

In the bottom row we should have the same relationship. In the left frame the arrow is
behind the circle. We can eliminate the 1st and 4th answer choices because they have
the arrow in front of the circle. We can eliminate the 2nd choice because the arrow
did not rotate, and the 3rd choice because the shapes did not swap colors.

We are left with the 5th choice as the correct answer.

165. Explanation
The correct answer is 4.
In the top row, every arrow in the left frame goes with a particular figure in the right
frame.
 Arrow pointing right = white heart
 Arrow pointing down = grey smiley face
 Arrow pointing up = grey sun
In the bottom row we should have the same relationship. We can eliminate the 3rd
answer choice because it contains a grey heart and not a white heart.
We can also eliminate the 1st and 5th choices because they contain frowny faces and
not smiley faces.
Finally we can eliminate the 2nd answer choice because we should have a smiley face
in the upper-right corner (where there was a down arrow in the left frame), and a sun
in the upper-left and lower-right corners (where in the left frame we had up arrows),
but we have the opposite.
We are left with the 4th choice as the only correct answer.

166. Explanation
The correct answer is 1.

There are two changes that occur between the first two figures in the first part of the
analogy: the figure rotates clockwise 90°, and the color changes from black to
white.

122
The same changes must apply to the third figure to complete the analogy.

This means that answer 1 is the correct answer.

167. Explanation
The correct answer is 3.

The relationship between the first two figures is as follows: from the first frame to the
second frame, all the circles become rectangles, and one of the small rectangles
disappears.
The correct answer must have the same relationship with the third figure: it should be
a large rectangle with three small rectangles.

This means that 3 is the correct answer.

168. Explanation
The correct answer is 2.
The relationship between the first two figures is as follows:
 The bottom shape (trapezoid) in the first figure becomes the outermost
shape in the second figure;
 The middle shape (triangle) is rotated 90° clockwise and becomes
the innermost shape the second figure;
 The top shape (circle) becomes the middle shape in the second figure.
The correct answer should have the same relationship with the third figure.
The bottom shape (rectangle) in the third figure must become the outermost shape.
This means we can eliminate answers 4 and 5.
The middle shape (triangle) must be rotated 90° and become the innermost shape, and
the top shape (circle) must become the middle shape. Answers 1 and 3 can be
eliminated, since they do not have the triangle in the correct position, leaving 2 as the
correct answer.

169. Explanation
The correct answer is 4.

The relationship between the first two figures is as follows:

- The inner shape (triangle) in the first figure moves to fill in the missing piece of
the outer shape.
- The inner shape (triangle) is rotated 90° counterclockwise.
- The inner shape (triangle) changes from white to black.

The correct answer will have the same relationship with the second figure. Since the
inner shape must move to the outside of the outer shape, we can eliminate answers 1
and 5. Answers 2 and 3 can also be eliminated since the outer shape is not supposed to

123
rotate.

This leaves 4 as the correct answer.

170. Explanation
The correct answer is 5.

The second figure is the same as first figure with two changes: the figure
becomes shaded, and the figure is extended in the direction of the dark area. The
correct answer should have the same relationship with the third figure.

This means that 5 is the correct answer.

171. Explanation
The correct answer is 2.

The relationship between the first two figures is as follows: the shapes in the first
frame each become shapes with one less side, the black shape becomes white and
the white shape becomes black. The correct answer must have the same relationship
with the third figure. The correct answer will be a large black triangle with a small
white four-sided shape.

This means that 2 is the correct answer.

172. Explanation
The correct answer is 4.

The relationship between the first two figures is as follows: the shapes in the first
frame each become shapes with one more side, the black shape becomes white and
the white shape becomes black. The correct answer must have the same relationship
with the third figure. The correct answer will be a large white pentagon with a small
black four-sided shape.

This means that 4 is the correct answer.

173. Explanation
The correct answer is 1.

The relationship between the first and second figure is as follows: the shape on the
left disappears, and the remaining shape on the right turns black. The correct
answer must have the same relationship with the third figure. This means that the
triangle on the left will disappear, and the remaining rectangle on the right will remain
and turn black.

124
This means that 1 is the correct answer.

174. Explanation
The correct answer is 3.

In the top row, from left to right, the lower half of the figure is removed. The same
should happen in the bottom row.
Since in the left frame we have two grey circles on top and two black circles on the
bottom, in the right frame we should only have the grey circles on top. We can
eliminate all the answer choices with black circles on the bottom (the 1st, 2nd, 4th,
and 5th choices).

We are left with the 3rd choice, which is the only correct answer.

175. Explanation
The correct answer is 3.

Look closely at the top row from left to right:


In the left frame there is a circle with a vertical and a horizontal line inside. In the
next frame (on the right), the circle shrinks and, in the center, overlaps the lines.

The same pattern should occur in the bottom row:


In the left frame, there is a diamond with two diagonal lines. In the next frame, the
diamond should shrink and overlap the diagonal lines in the center of the frame.

Answer choice 1 is incorrect as the diamond is transparent and the lines can be seen.
Answer choice 2 is incorrect since the lines are no longer diagonal and the shape in
the middle is a circle instead of a diamond.
Answer choice 4 is incorrect since the lines are not diagonal and there is no shape in
the center.
Answer choices 5 is incorrect as it contains a black (not grey) diamond.

We are left with the 3rd choice, which is the only correct answer.

176. Explanation
The correct answer is 1.

Take a look at the top row. The order of the shapes in the left frame determines the
position of the shapes in the right frames.
 The top shape (the star) is positioned in the center,
 The middle shape (the circle) is positioned on the outside, and -
 The bottom shape (the triangle) is positioned in between the two objects.

125
The same rule will apply in the bottom row:
The circle sector will be in the center, so we can eliminate answer choices 4 and 5.
The plaque will be the outer shape, so we can eliminate answer choice 2.
The ring will be positioned in between both shapes. We can eliminate answer choice 3,
and are left with answer choice 1 as the correct answer.

126

You might also like